0% found this document useful (0 votes)
27 views95 pages

Applied Math I Lecture Notes 3 (Chapters 1-3)

Uploaded by

ascodej
Copyright
© © All Rights Reserved
We take content rights seriously. If you suspect this is your content, claim it here.
Available Formats
Download as PDF, TXT or read online on Scribd
0% found this document useful (0 votes)
27 views95 pages

Applied Math I Lecture Notes 3 (Chapters 1-3)

Uploaded by

ascodej
Copyright
© © All Rights Reserved
We take content rights seriously. If you suspect this is your content, claim it here.
Available Formats
Download as PDF, TXT or read online on Scribd
You are on page 1/ 95

Debre Berhan University

Applied mathematics One(Math 1041)


Lecture note: version 1.1
”Mathematics is the most beautiful and most powerful creation of
the human spirit.”
Stefan Banach

Dejen Ketema
Department of Mathematics
[email protected]

March, 2020
Contents

1 Vectors and vector spaces 2


1.1 Geometry and Distance . . . . . . . . . . . . . . . . . . . . . . . . . . . . . . . . 2
1.2 Vectors & dot product . . . . . . . . . . . . . . . . . . . . . . . . . . . . . . . . 7
1.3 Direction angles and direction cosine . . . . . . . . . . . . . . . . . . . . . . . . 18
1.4 Cross Product . . . . . . . . . . . . . . . . . . . . . . . . . . . . . . . . . . . . . 19
1.5 Lines and planes in space . . . . . . . . . . . . . . . . . . . . . . . . . . . . . . . 24
1.6 Vector space and Subspace . . . . . . . . . . . . . . . . . . . . . . . . . . . . . . 31
1.7 Linear dependence, independence and Basis . . . . . . . . . . . . . . . . . . . . 35

1
Chapter 1

Vectors and vector spaces

1.1 Geometry and Distance


The collection of all real numbers is denoted by R. Thus R includes the integers

· · · , −2, −1, 0, 1, 2, · · ·

the
√ rational numbers, p/q, where p and q are integers (q = 6 0), and the irrational numbers, like
2, π, e, etc. Members of R may be visualized as points on the real-number line as shown in

We write a ∈ R to mean a is a member of the set R. In other words, a is a real number.


Given two real numbers a and b with a < b, the closed interval [a, b] consists of all x such that
a ≤ x ≤ b, and the open interval (a, b) consists of all x such that a < x < b. Similarly, we may
form the half-open intervals [a, b) and (a, b]. The absolute value of a number a ∈ R is written
as |a| and is defined as 
a if a ≥ 0
|a| = 
−a if a < 0

Definition 1.1
A point in the plane R2 has two coordinates P = (x, y). A point in space R3 is
determined by three coordinates P = (x, y, z). The signs of the coordinates define 4
quadrants in R2 or 8 octants in R3 . These regions intersect at the origin O = (0, 0)
or O = (0, 0, 0) and are bound by coordinate axes {y = 0} and {x = 0} or coordinate
planes {x = 0}, {y = 0}, {z = 0}.

In R2 its custom to orient the x-axis to the ”east” and the y-axis ”north”. In R3 , the most
common coordinate system has the xy-plane as the ”ground” and the z-coordinate axes pointing
”up”.

2
1.1. GEOMETRY AND DISTANCE DBU

Example 1.1

1. P = (−2, −3) is in the third quadrant of the plane and P = (1, 2, 3) is in the
positive octant of space. The point (0, 0, −8) is located on the negative z axis.
The point P = (1, 2, −3) is below the xy-plane. Can you spot the point Q on the
xy-plane which is closest to P ?

2. Problem. Find the midpoint M of P = (7, 2, 5) and Q = (−15, 4, 7). Answer.


The midpoint is the average of each coordinate M = (P + Q)/2 = (−4, 3, 6).

3. In computer graphics or photography, the xy-plane represents the retina or film


plate and the z-coordinate measures the distance towards the viewer. In this pho-
tographic coordinate system, your eyes and chin define the plane z = 0 and the
nose points in the positive z direction. If the midpoint of your eyes is the origin of
the coordinate system and your eyes have the coordinates (1, 0, 0), (−1, 0, 0), then
the tip of your nose might have the coordinates (0, −1, 1).

Definition 1.2
The Euclidean distance
q between two points P = (x, y, z) and Q = (a, b, c) in space is
defined as d(P, Q) = (x − a)2 + (y − b)2 + (z − c)2 .

Example 1.2

Problem: Find the distance d(P, Q) between P = (1, 2, 5) and Q = (−3, 4, 7) and
verify
√ that d(P, M
√) + d(Q, M ) = d(P, Q). Answer:
√ The distance
√ is d(P, Q) =
2 2
4 + 2 +√ 2 2 2 2
2 = 24. The√distance d(P, M ) is 2 + 1 + 1 = 6. The distance
d(Q, M ) is 22 + 12 + 12 = 6. Indeed d(P, M ) + d(M, Q) = d(P, Q).

Remarks.
1. A distance can be defined by taking any non-negative function d(P, Q) = d(Q, P ) which
satisfies the triangle inequality d(P, Q)+d(Q, R) ≥ d(P, R) and which has the property
that d(P, Q) = 0 if and only if P = Q. A set X equipped with such a distance function
d is called a metric space.
2. Symmetry distinguishes the Euclidean distance from other distances. It is characterized
by the property d((1, 0, 0), (0, 0, 0)) = 1 together with the requirement of rotational and
translational and scaling symmetry d(λP, λQ) = λd(P, Q).
3. We usually work with a right handed coordinate system, where the x, y, z axes
can be matched with the thumb, pointing and middle finger of the right hand. The
photographers coordinate system is an example of a left handed coordinate system,
where we use the thumb and pointing finger and middle finger of the left hand. Nature
is not oblivious to parity. Some fundamental laws in particle physics related to the weak
force are different when observed in a mirror. Coordinate systems with different parity
can not be rotated into each other.
4. When dealing with
q geometric problems in the plane, we leave the z-coordinate away and
have d(P, Q) = (x − a)2 + (y − b)2 , where P = (x, y), Q = (a, b). Its important to work
in R2 without referring to a possible R3 in which it might be embedded.

c Dejen K. 2020 3
1.1. GEOMETRY AND DISTANCE DBU

Points, curves, surfaces and solids are geometric objects which can be described with
functions of several variables. An example of a curve is a line, an example of a surface is
a plane, an example of a solid is the interior of a sphere. We focus next on spheres or circles.
Definition 1.3
A circle of radius r ≥ 0 centered at P = (a, b) is the collection of points in R2 which
have distance r from P .

Definition 1.4
A sphere of radius ρ centered at P = (a, b, c) is the collection of points in R3 which have
distance ρ ≥ 0 from P . The equation of a sphere is (x − a)2 + (y − b)2 + (z − c)2 = ρ2 .

Definition 1.5
An ellipse is the collection of points P in R2 for which the sum d(P, A) + d(P, B) of the
distances to two points A, B is a fixed constant l > 0 larger than d(A, B). This allows
to draw the ellipse with a string of length l attached at A, B. When 0 is the midpoint of
2 2
A, B, an algebraic description is the set of points which satisfy the equation xa2 + yb2 = 1.

Example 1.3

1. Problem: Is the point (3, 4, 5) outside or inside the sphere (x − 2)2 + (y − 6)2 +
(z − 2)2 = 16? Answer: The distance of the point to the center of the sphere is

1 + 4 + 9. Since this is smaller than 4 the radius of the sphere, the point is inside.

2. Problem: Find an algebraic expression for the set of all points for which the sum
of the distances
q to A = (1,q0) and B = (−1, 0) is equal to 3. Answer: Square the
equation (x − 1)2 + y 2 + (x + 1)2 + y 2 = 3, separate the remaining single square
root on one side and square again. Simplification gives 20x2 + 36y 2 = 45 which is
2 2
equivalent to xa2 + yb2 = 1, where a, b can be computed as follows: because P = (a, 0)
satisfies this equation, d(P, A) + d(P, B) = (a − 1) + (a + 1) = 3 so that √ a = 3/2.
Similarly,
√ the point Q = (0, b) satisfying it gives d(Q, A) + d(P, B) = 2 b2 + 1 = 3
or b = 5/2.

Definition 1.6
When completing the square of an equation x2 + bx + c = 0, we add (b/2)2 − c on
both sides ofqthe equation in order to get (x + b/2)2 = (b/2)2 − c. Solving for x gives
x = −b/2 ± (b/2)2 − c.

c Dejen K. 2020 4
1.1. GEOMETRY AND DISTANCE DBU

Example 1.4

1. The equation 2x2 − 10x + 12 = 0 is equivalent to x2 + 5x = −6. Adding (5/2)2 on


both sides gives (x + 5/2)2 = 1/4 so that x = 2 or x = 3.

2. The equation x2 + 5x + y 2 − 2y + z 2 = −1 is after completion of the square (x +


5/2)2 − 25/4 + (y − 1)2 − 1 + z 2 = −1 or (x − 5/2)2 + (y − 1)2 + z 2 = (5/2)2 . We
see a sphere center (5/2, 1, 0) and radius 5/2.

c Dejen K. 2020 5
1.1. GEOMETRY AND DISTANCE DBU

Exercise 1.1

1. (a) a) Find its center and radius of the sphere S given by x2 + y 2 + z 2 − 20x +
12y + 4z = 29.
(b) Find the distance from the center of S defined in a) to x2 + y 2 + z 2 = 900.
(c) Find the minimal distance between the spheres. This is the minimal distance
between two points where each is in one sphere.

2. (a) Find the distance a from P = (−12, −4, −6) to x = 0.


(b) Find the distance b from P to the x-axes.
(c) Find the distance c from P to the origin O = (0, 0, 0).
(d) Take a general point P = (x, y, z) and repeat the computation for the values
a, b, c as in a − c). What is ar + b2 − c2 ?

3. (a) Find an equation of the largest sphere with center (4, 11, 9) that is contained
in the first octant {x ≤ 0, y ≤ 0, z ≥ 0}.
(b) Find the equation for the sphere centered at (6, 10, 8) which passes through
the center (4, 11, 9)of the sphere in a).

4. (a) Describe the surface x2 − (z 2 + 4z) = 12 in R3 .


(b) What is the surface x2 = z 2 in three dimensional space R3 .
(c) Draw the surfaces of a) and b) and their intersection.

5. You play billiard in the table {(x, y)|0 ≤ x ≤ 4, 0 ≤ y ≤ 8}.

(a) Hit the ball at (3, 2) to reach the hole (4, 8) bouncing 3 times at the left wall
and three times at the right wall and no other walls. Find the length of the
shot.
(b) Hit from (3, 2) to reach the hole (4, 0) after hitting twice the left and twice
the right wall as well as the top wall y = 8 once. What is the length of the
trajectory?

6. Verify that the radius of the inscribed circle in a 3 : 4 : 5 triangle is 1. Use the
picture with the triangle ABC given by:

A = (0, 0), B = (4, 0), C = (0, 3), introduce M = (1, 1) then get the coordinates of
the points X, Y, Z, then compute the distances.
c Dejen K. 2020 6
1.2. VECTORS & DOT PRODUCT DBU

Definition 1.7
A physical quantity is a physical property of a phenomenon, body, or substance, that can
be quantified by measurement.

Definition 1.8
Quantities which can be described by magnitude only are called scalars. Scalars are
represented by a single letter, such as a. Some examples of scalar quantities are mass
(five kilograms), temperature (twenty-two degrees Celsius), and numbers without units
(such as three).

1.2 Vectors & dot product


Geometric view :
Definition 1.9
A vector is defined as having a magnitude and a direction. We represent it by an arrow in
the plane or in space. The length of the arrow is the vector’s magnitude and the direction
of the arrow is the vector’s direction.

In this way, two arrows with the same magnitude and direction represent the same vector.

(same vector)

We will refer to the start of the arrow as the tail and the end as the tip or head.

P~Q

To distinguish vectors from points, it is custom to write [2, 3, 4]T or h2, 3, 4i for vectors and
(2, 3, 4) for points.

2A
A -A (scaling a vector)
1
2
A

c Dejen K. 2020 7
1.2. VECTORS & DOT PRODUCT DBU

Because we use numbers to scale a vector we will often refer to real numbers as scalars. You
add vectors by placing them head to tail. As the figure shows, this can be done in either order

A+B
A A

It is often useful to think of vectors as displacements. In this way, A + B can be thought of


as the displacement A followed by the displacement B.
You subtract vectors either by placing the tail to tail or by adding A + (−B).

A-B

B
A

Thought of as displacements A − B is the displacement from the end of B to the end of A.

Algebraic view:
Definition 1.10
 
x−a
Two points P = (a, b, c) and Q = (x, y, z) in R3 define a vector ~v =  y − b  which we
 

z−c
T
simply write also as [x−a, y −b, z −c] or hx−a, y −b, z −ci to save space. It is custom in
linear algebra to call [3, 4, 5] a row vector and its transpose [3, 4, 5]T = h3, 4, 5i a column
vector. As it goes from P to Q we write ~v = P~Q. The real numbers numbers p, q, r in
~v = [p, q, r]T are called the components of ~v .

Vectors can be drawn anywhere in space. But two vectors with the same components are
considered equal. Vectors can be translated into each other if their components are the same.
If a vector ~v starts at the origin O = (0, 0, 0), then ~v = [p, q, r]T heads to the point (p, q, r).
One can therefore identify points P = (a, b, c) with vectors ~v = [a, b, c]T attached to the origin.
For clarity, we often draw an arrow on top of vectors and if ~v = P~Q then P is the ”tail” and
Q is the ”head” of the vector.
Definition 1.11

The sum of two vectors is A ~+B~ = [a1 , a2 ]T + [b1 , b2 ]T = [a1 + b1 , a2 + b2 ]T . The scalar
multiple is λA ~ = λ[a1 , a2 ] = [λa1 , λa2 ]T . The difference A
T ~−B ~ can best be seen as
~ and (−1)B.
the addition of A ~

c Dejen K. 2020 8
1.2. VECTORS & DOT PRODUCT DBU

On can check commutativity, associativity, or distributivity rules for vectors as for numbers.
Definition 1.12

The vectors ~i = [1, 0]T , ~j = [0, 1]T are called the standard basis vectors in the plane.
In space, one has the standard basis vectors ~i = [1, 0, 0]T , ~j = [0, 1, 0]T , ~k = [0, 0, 1]T .

Every vector ~v = [p, q]T in the plane can be written as a combination ~v = p~i + q~j of standard
basis vectors. Every vector ~v = [p, q, r]T in space can be written as ~v = p~i + q~j + r~k. Vec-
tors appear everywhere in applications. For example in mechanics: if r(t) ~ = [f (t), g(t)]T is a
point in the plane which depends on time t, then ~v = [f 0 (t), g 0 (t)]T is the velocity vector at
~ Here f 0 (t), g 0 (t) are the derivatives. In physics, where we often want to determine forces
r(t).
acting on objects, these forces are represented as vectors. In particular, electromagnetic or
gravitational fields or velocity fields in fluids are described by vectors. Vectors appear also in
computer science: the ”scalable” vector graphics ”format” is a standard for the web for describ-
ing graphics. In quantum computation, rather than working with bits, one deals with qbits,
which are vectors. Finally, color can be written as a vector ~v = [r, g, b]T , where r is red, g is
green and b is blue component of the color vector. An other coordinate system for color is
~v = [c, m, y]T = [1 − r, 1 − g, 1 − b]T , where c is cyan, m is magenta and y is yellow. Vectors
appear also in probability theory and statistics. On a finite probability space for example, a
random variable is nothing else than a vector.

The addition and scalar multiplication of vectors satisfy the laws known from arithmetic.

1. commutativity ~u + ~v = ~v + ~u,

2. associativity ~u + (~v + w) ~ r ∗ (s ∗ ~v ) = (r ∗ s) ∗ ~v
~ = (~u + ~v ) + w,

3. distributivity (r + s)~v = ~v (r + s)

4. And r(~v + w)
~ = r~v + rw,
~ where * denotes multiplication with a scalar.

Definition 1.13

The length |~v | of a vector ~v = P~Q is defined as the distance d(P, Q) from P to Q. A
vector of length 1 is called a unit vector. If ~v 6= ~0, then |~~vv| is called a unit vector.

c Dejen K. 2020 9
1.2. VECTORS & DOT PRODUCT DBU

Example 1.5

|[3, 4]T | = 5 and |[3, 4, 12]T | = 13. Examples of unit vectors are |~i| = |~j| = |~k| = 1
and [3/5, 4/5]T and [3/13, 4/13, 12/13]T . The only vector of length 0 is the zero vector
|~0| = 0.

Example 1.6

1. Add the following two vectors

(a) ~u = (−1, 2, 1) and ~v = (2, −5, 0)


(b) ~u = i + 4j − 2k and ~v = 2i − 5j

2. Let ~u = 3i + 4j − k, then find


2
(a) ~u
3
(b) −1~u

solution:

1. (a)

~u + ~v = (−1, 2, 1) + (2, −5, 0)


= (−1 + 2, 2 − 5, 1 + 0)
= (1, −3, 1)

(b)

~u + ~v = (i + 4j − k) + (2i − 5j)
= (1 + 2)i + (4 − 5)j + (−1 + 0)k
= 3i − j − k

2. (a)
2 2
~u = (3i + 4j − k)
3 3
8 2
= 2i + j − k
3 3

(b)

−1~u = −(3i + 4j − k)
= −3i − 4j + k

c Dejen K. 2020 10
1.2. VECTORS & DOT PRODUCT DBU

Dot Product
The dot product is one way of combining (”multiplying”) two vectors. The output is a scalar
(a number). It is called the dot product because the symbol used is a dot. Because the dot
product results in a scalar it, is also called the scalar product.
The geometric definition of the dot product says that the dot product between two vectors ~a
and ~b is
~a · ~b = k~ak · k~bk cos θ,
where θ is the angle between vectors ~a and ~b. Although this formula is nice for understanding
the properties of the dot product, a formula for the dot product in terms of vector components
would make it easier to calculate the dot product between two given vectors.

Since the standard unit vectors are orthogonal, we immediately conclude that the dot product
between a pair of distinct standard unit vectors is zero:

i · j = i · k = j · k = 0.

The dot product between a unit vector and itself is also simple to compute. In this case, the
angle is zero and cos θ = 1. Given that the vectors are all of length one, the dot products are

i · i = j · j = k · k = 1.

The second step is to calculate the dot product between two three-dimensional vectors

a = (a1 , a2 , a3 ) = a1 i + a2 j + a3 k
b = (b1 , b2 , b3 ) = b1 i + b2 j + b3 k.

To do this, we simply assert that for any three vectors ~a, ~b, and ~c, and any scalar λ,

(λa) · b = λ(a · b)

(a + b) · c = a · c + b · c.

Definition 1.14
The dot product of two vectors ~v = [a, b, c]T and w
~ = [p, q, r]T is defined as ~v · w
~ =
ap + bq + cr.

Remarks.

1. Different notations for the dot product are used in different mathematical fields. While
mathematicians write ~v · w ~ T is used in quantum
~ the Dirac notation [~v |w]
~ = (~v , w),
i
mechanics or the Einstein notation vi w or more generally gij vi wj in general relativity.
The dot product is also called scalar product or inner product.

2. Any product g(v, w) which is linear in v and w and satisfies the symmetry g(v, w) =
g(w, v) and g(v, v) ≤ 0 and g(v, v) = 0 if and only if v = 0 can be used as a dot product.
An example is g(v, w) = 3v1 w1 + 2v2 w2 + v3 w3 .

c Dejen K. 2020 11
1.2. VECTORS & DOT PRODUCT DBU

Theorem 1.1
The dot product determines distances and distances determines the dot product.

Proof:
√ Write v = ~v . Using the dot product one can express the length of v as |v| =
v · v. On the other hand, from (v + w) · (v + w) = v · v + w · w + 2(v · w) can be solved
for v · w:
v · w = (|v + w|2 − |v|2 − |w|2 )/2.

Theorem 1.2
The Cauchy-Schwarz inequality tells |~v · w|
~ ≤ |~v ||w|.
~

Proof. If |w| = 0, there is nothing to show. Otherwise, assume |w| = 1 by dividing the
equation by |w|. Now plug in a = v · w into the equation 0 ≤ (v − aw) · (v − aw) to get
0 ≤ (v − (v · w)w) · (v − (v · w)w) = |v|2 + (v · w)2 − 2(v · w)2 = |v|2 − (v · w)2 which means
(v · w)2 ≤ |v|2 .

Having established this, it is possible to give a definition of what an angle is, without referring
to any geometric pictures:
Definition 1.15
The angle between two nonzero vectors ~v , w ~ is defined as the unique α ∈ [0, π] which
satisfies ~v · w
~ = |~v | · |w|
~ cos(α). Since cos maps [0, π] in a 1 : 1 manner to [−1, 1], this is
well defined.

Note:
1. The cosine of the angle between two non-zero vectors ~a and ~b is
~a · ~b
cos θ =
k~akk~bk
 
~a · ~b 
=⇒ θ = cos−1 
k~akk~bk
π
2. If an angle between two vectors ~a and ~b is then ~a · ~b = 0 and the vectors ~a and ~b are
2
known as orthogonal vectors. In general the dot product of two non-zero vectors is zero
only if they are orthogonal.

c Dejen K. 2020 12
1.2. VECTORS & DOT PRODUCT DBU

Remark:- Suppose ~a and ~b are two non-zero vectors and θ is the angle between ~a and ~b:

i) if θ is acute, then ~a · ~b > 0.

ii) if θ is obtuse, then ~a · ~b < 0.

iii) if θ is right angle, then ~a · ~b = 0.

Theorem 1.3: Al Kashi’s theorem:


If a, b, c are the side lengths of a triangle ABC and α is the angle at the vertex C, then
a2 + b2 = c2 − 2ab cos(α).

Proof. Define ~v = AB, ~ w ~ = AC. ~ Because c2 = |~v − w|~ 2 = (~v − w) ~ · (~v − w) ~ =


2 2
|~v | + |w|
~ − 2~v w, ~ cos(α) so that c = |~v |2 + |w|
~ = |~v | · |w|
~ We know ~v w 2
~ 2 − 2|~v | · |w|
~ cos(α) =
a2 + b2 − 2ab cos(α).

The angle definition works in any space with a dot product. In statistics one works with
vectors of n components. They are called data or random variables and cos(α) is called the
correlation between two random variables ~v , w ~ of zero expectation E[~v ] = (v1 + · · √
· + vn )/n.
The dot product (v1 w1 + · · · + vn wn )/n is then the covariance, the scaled length |v|/ n is the
standard deviation and denoted by σ(v). The formula Corr[v, w] = Cov[v, w]/(σ(v)σ(w))
for the correlation is the familiar angle formula. Statistics shows that geometry in arbitrary
dimensions can be useful.
Theorem 1.4
The triangle inequality tells |~u + ~v | ≤ |~u| + |~v |.
Proof: |~u +~v |2 = (~u +~v )·(~u +~v ) = ~u2 +~v 2 +2~u ·~v ≤ ~u2 +~v 2 +2|~u ·~v | ≤ ~u2 +~v 2 +2|~u|·|~v | =
(|~u| + |~v |)2 .

Example 1.7

Calculate the dot product of a = (1, 2, 3) and b = (4, −5, 6). Do the vectors form an
acute angle, right angle, or obtuse angle?
Solution: Using the component formula for the dot product of three-dimensional vectors,

a · b = a1 b 1 + a2 b 2 + a3 b 3 ,

we calculate the dot product to be

a · b = 1(4) + 2(−5) + 3(6) = 4 − 10 + 18 = 12.

Since a.b is positive, we can infer from the geometric definition, that the vectors form an
acute angle.

c Dejen K. 2020 13
1.2. VECTORS & DOT PRODUCT DBU

Example 1.8

Calculate the dot product of c = (−4, −9) and d = (−1, 2) . Do the vectors form an
acute angle, right angle, or obtuse angle?

Solution: Using the component formula for the dot product of two-dimensional vectors,

a · b = a1 b1 + a2 b2 ,

we calculate the dot product to be

c · d = −4(−1) − 9(2) = 4 − 18 = −14.

Since c.d is negative, we can infer from the geometric definition, that the vectors form an
obtuse angle.

Example 1.9

If a = (6, −1, 3), for what value of c is the vector b = (4, c, −2) perpendicular to a?

Solution: For a and b to be perpendicular, we need their dot product to be zero. Since

a · b = 6(4) − 1(c) + 3(−2) = 24 − c − 6 = 18 − c,

the number c must satisfy 18 − c = 0, or c = 18. You can double-check that the vector
b = (4, 18, −2) is indeed perpendicular to a by verifying that

a · b = (6, −1, 3).(4, 18, −2) = 0.

1. Find the value of k such that the vectors (2, 3, −2) and (5, k, 2) are perpendicular.

2. Find the values of β such that the vectors (β, −3, 1) and (β, β, 2) are perpendicular.

c Dejen K. 2020 14
1.2. VECTORS & DOT PRODUCT DBU

Example 1.10

1. Find the component form and the length of vector with initial point and terminal
point p = (3, −7, 1) and Q = (−2, 5, 3)

2. Find the unit vector in the direction of ~v = (−2, 5, 3)

3. Let ~u = (2, 5, −1) and ~v = (−1, 2, 3),the find the scalar product of ~u and ~u

Solution

1. ~v = P~Q = (−2
q − 3, 5 + 7, 3 − 1) =√
(−5, 12, 2)
and k~v k = (−5)2 + (12)2 + 22 = 173

√ √
2. k~v k = −22 + 52 + 32 = 38
1
û = k~~vvk = √ (−2, 5, 3)
38
−2 5 3
= (√ , √ , √ )
38 38 38

3. ~u · ~v = (2, 5, −1) · (−1, 2, 3)


= 2. − 1 + 5.2 + −1.3
= −2 + 10 − 3
=5

Exercise 1.2

1. Find the cosine of the angle and the angle between the following vectors

(a) ~u = 2i − 5j + k and ~v = i + j + 2k

(b) ~v = (− 3, 1) and w~ = (0, 5)
(c) ~v = (0, −1, 0) and w
~ = (0, 0, 3)

Definition 1.16
Two vectors are called orthogonal or perpendicular if ~v · w ~ = 0. The zero vector ~0 is
T
~ = [−3, 2]T .
orthogonal to any vector. For example, ~v = [2, 3] is orthogonal to w

Theorem 1.5
Pythagoras theorem: if ~v and w ~ are orthogonal, then |v − w|2 = |v|2 + |w|2 .
Proof: (~v − w)
~ · (~v − w)
~ = ~v · ~v + w
~ ·w
~ + 2~v · w
~ = ~v · ~v + w
~ · w.
~

Projection
Suppose that two non-zero vectors ~v and w ~ are positioned as in the figure below and that the
light casts a shadow on the line containing the vector ~v . Informally we think of the shadow as

c Dejen K. 2020 15
1.2. VECTORS & DOT PRODUCT DBU

determining a vector parallel to vector w,


~ we call this vector the projection of vector w
~ on to
w w v·w
vector ~v which is denoted by projv and defined as projv = ( kvk2 )v.

Definition 1.17
~v · w
~
The vector P (~v ) = ~ is called the projection of ~v onto w.
w ~ The scalar projection
|w|
~2
~v · w
~
is a signed length of the vector projection. Its absolute value is the length of the
|w|
~
~ The vector ~b = ~v − P (~v ) is a vector orthogonal to the w-direction.
projection of ~v onto w. ~

Example 1.11

T
1. For example, ~ = [1, −1, 0]T , P (~v ) = [1/2, −1/2, 0]T . Its
√ with ~v = [0, −1, 1] , w
length is 1/ 2.

2. Projections are important in physics. If a wind force F~ affects a car driving in the
direction w
~ and P denotes the projection onto w ~ then Pw~ (F~ ) is the force which
accelerates or slows down the car.

The projection allows to visualize the dot product. The absolute value of the dot product
is the length of the projection. The dot product is positive if v points more towards to w,
it is negative if v points away from it. In the next lecture we use the projection to compute
distances between various objects.

c Dejen K. 2020 16
1.2. VECTORS & DOT PRODUCT DBU

Example 1.12

1. Let vector ~u = 2i − j + 3k, ~v = i + 2j + k, then find:


~
u ~
u
(a) proj~v (d) The scalar projection of proj~v and
(b) proj~u ~v proj~u~v
~
3u
(c) proj 1~v
2

Solution:-

1. (a) (c)
 
~v · ~u 1
proj~v~u = v · 3~u 1

~v u 2~
k~v k2 proj 3~
1 = ~v
  2~v
k 12 ~v k2 2
(i + 2j + k) · (2i − j + 3k) 1
= (i + 2j + k) 2 (i + 2j + k) · 3(2i − j + 3k) 1
ki + 2j + kk2 =( ) (i + 2j + k)
1 2 2
2−2+3 4 ki + 2j + kk
=( 2 )(i + 2j + k) 3
1 + 22 + 12 2 (2 − 2 + 3) 1
3 =( 1 2 2 2
) (i + 2j + k)
= (i + 2j + k) 4 (1 + 2 + 1 )
2
6
1 1 = 3(i + 2j + k)
= i+j+ k = 3i + 6j + 3k
2 2
(b)
 
~u · ~v
proj~u~v = ~u
k~uk2
(2i − j + 3k) · (i + 2j + k)
=( 2 )(2i − j + 3k)
k2i − j + 3kk
2−2+3
=( 2 )(2i − j + 3k)
1 + (−2)2 + 32
3
= (2i − j + 3k)
14
6 3 9
= i− j+ k
14 14 14

2.
~v · ~u
kproj~v ~u k = |( )|
k~v k
(i + 2j + k) · (2i − j + 3k)
= |( )|
ki + 2j + kk
2−2+3
= |( √ )|
12 + 2 2 + 1 2
3
=√
6
~
v ~u · ~v
&kproj~u k = |( )|
k~uk
(2i − j + 3k) · (i + 2j + k)
= |( )|
k2i − j + 3kk
2−2+3
= |p |
1 + (−2)2 + 32
2

3

14

c Dejen K. 2020 17
1.3. DIRECTION ANGLES AND DIRECTION COSINE DBU

1.3 Direction angles and direction cosine


Definition 1.18
Let p be a vector and α, β and γ are angles that a vector p makes with the positive x, y
and z axis respectively. These angles are called directional angles of the vector p and
cos α, cos β and cos γ are directional cosines of vector

Figure 1.1: direction angeles

Note:-
y √
1. cos α = xr , cos β = r
and cos γ = z
r
where r = x2 + y 2 + z 2 .

2. cos2 α + cos2 β + cos2 γ = 1

Example 1.13

1. consider the following vectors

(a) ~a = (2, 3)
(b) ~b = (1, 1, 1)
(c) ~c = (3, 5, 2)

Then find the direction angles and direction cosines


π
2. If the direction angle of a vector ~a makes an angle of with the positive x-axis and
3
π
an angle of with the positive y-axis then what angle will make with the positive
4
z-axis.

c Dejen K. 2020 18
1.4. CROSS PRODUCT DBU

Exercise 1.3

~ if ~u = [−10, 2, 9]T and ~v = [1, 1, 3]T and


1. Find a unit vector parallel to ~u + 2~v + 4w
~ = [3, 1, 1]T .
w

2. A Alemitu gets pulled with a force F~ = h7, 1, 4i. She moves with velocity ~v =
h4, −2, 1i. The dot product of F~ with ~v is power.

(a) Find the angle between the F~ and ~v .


(b) Find the vector projection of the F~ onto ~v .
(c) Find the scalar projection of F~ on ~v .

3. Light shines long the vector ~a = ha1 , a2 , a3 i and reflects at the three coordinate
planes where the angle of incidence equals the angle of reflection. Verify that the
reflected ray is −~a. Hint. Reflect first at the xy-plane.

4. A rope is wound exactly 5 times around a stick of circumference 1 and length 12.
How long is the rope?

1.4 Cross Product


Definition 1.19
The cross product of two vectors ~v = [v1 , v2 ]T and w
~ = [w1 , w2 ]T in the plane is the scalar
v1 w2 − v2 w1 .

To remember this, write it as a determinant of a matrix A which is a 2×2 array of the numbers).
det(A) is the product of the diagonal entries minus the product of the side diagonal entries.
" #
v1 v2
.
w1 w2

Definition 1.20
The cross product of two vectors ~v = [v1 , v2 , v3 ]T and w ~ = [w1 , w2 , w3 ]T in space is
~ = [v2 w3 − v3 w2 , v3 w1 − v1 w3 , v1 w2 − v2 w1 ]T .
defined as the vector ~v × w

To remember it we write the product as a ”determinant”:


       
i j k i j k
 v1 v2 v3  = 
  
v2 v3  −  v1
 
v3  +  v1 v2
  

w1 w2 w3 w2 w3 w1 w3 w1 w2

c Dejen K. 2020 19
1.4. CROSS PRODUCT DBU

which is ~i(v2 w3 − v3 w2 ) − ~j(v1 w3 − v3 w1 ) + ~k(v1 w2 − v2 w1 ).


Example 1.14

1. The cross product of [1, 2]T and [4, 5]T is 5 − 8 = −3.

2. The cross product of [1, 2, 3]T and [4, 5, 1]T is [−13, 11, −3]T .

Theorem 1.6
In space, the cross product ~v × w
~ is orthogonal to both ~v and w.
~ The product is anti-
commutative.
Proof. We verify for example that ~v · (~v × w)
~ = 0 and look at the definition.

Theorem 1.7
Length formula for the cross product: |~v × w|
~ = |~v ||w|
~ sin(α).
Proof: verify first the Lagrange’s identity |~v × w| ~ 2 = |~v |2 |w|
~ 2 − (~v · w)
~ 2 which is also
called Cauchy-Binet formula by direct computation . Now, |~v ·~w| = |~v ||w| ~ cos(α).

Definition 1.21
The absolute value respectively length |~v × w|
~ defines the area of the parallelogram
spanned by ~v and w.
~

Note that we have given the definition of area, so that nothing needs to be proven. To see
that the definition fits with our common intuition we have about area, note that |w| ~ sin(α) is
the height of the parallelogram with base length |~v |. The area formula proves the sin-formula
because the area does not depend on which pair of sides to a triangle we take. The area makes
sense because it is linear in each of the vectors ~v and w:
~ scale one by a factor λ = 2 for example,

c Dejen K. 2020 20
1.4. CROSS PRODUCT DBU

doubles the area.

Theorem 1.8
~v × w
~ is zero if and only if ~v and w
~ are parallel, that is if ~v = λw
~ for some real λ.
Proof. Use the sin formula and the fact that sin(α) = 0 if α = 0 or α = π. The cross
product can therefore be used to check whether two vectors are parallel or not. Note that
v and −v are considered parallel even so sometimes the notion anti-parallel is used.

Theorem 1.9
The trigonometric sin-formula: if a, b, c are the side lengths of a triangle and α, β, γ are
the angles opposite to a, b, c then a/ sin(α) = b/ sin(β) = c/ sin(γ).
Proof. Express the area of the triangle in three different ways:

ab sin(γ) = bc sin(α) = ac sin(β).

Divide the first equation by sin(γ) sin(α) to get one identity. Divide the second equation
by sin(α) sin(β) to get the second identity.

Example 1.15

If ~v = [a, 0, 0]T and w


~ = [b cos(α), b sin(α), 0]T , then v × w
~ = [0, 0, ab sin(α)]T which has
length |ab sin(α)|.

Definition 1.22
~ = ~u · (~v × w)
The scalar [~u, ~v , w] ~ is called the triple scalar product of ~u, ~v , w.
~

Definition 1.23
The absolute value of [~u, ~v , w]
~ defines the volume of the parallelepiped spanned by
~u, ~v , w.
~

Definition 1.24
The orientation of three vectors is defined as the sign of [~u, ~v , w].
~ It is positive if the
three vectors define a right-handed coordinate system.

Again, there was no need to prove anything because we defined volume and orientation. Why
does this fits with our intuition? The value h = |~u · ~n|/|~n| is the height of the parallelepiped

c Dejen K. 2020 21
1.4. CROSS PRODUCT DBU

if ~n = (~v × w)~ is a normal vector to the ground parallelogram of area A = |~n| = |~v × w|. ~ The
volume of the parallelepiped is hA = (~u · ~n/|~n|)|~v × w| ~ which simplifies to ~u · ~n = |(~u · (~v × w)|
~
which is the absolute value of the triple scalar product. The vectors ~v , w ~ and ~v × w ~ form a
right handed coordinate system. If the first vector ~v is your thumb, the second vector w ~
is the pointing finger then ~v × w ~ is the third middle finger of the right hand. For example, the
vectors ~i, ~j,~i × ~j = ~k form a right handed coordinate system.

Since the triple scalar product is linear with respect to each vector, we also see that volume
is additive. Adding two equal parallelepipeds together for example gives a parallelepiped with
twice the volume.

c Dejen K. 2020 22
1.4. CROSS PRODUCT DBU

Example 1.16

1. Problem: You have two apples of the same shape, but one has a 3 times larger
diameter. What is their weight ratio?

Answer. For a parallelepiped spanned by [a, 0, 0]T [0, b, 0]T and [0, 0, c]T , the vol-
ume is the triple scalar product abc. If a, b, c are all tripled, the volume gets multi-
plied by a factor 27. Now cut each apple into the same amount of parallelepipeds,
the larger one with slices 3 times as large too. Since each of the pieces has 27 times
the volume, also the apple is 27 times heavier!

2. Problem: Find the volume of the parallelepiped which has the vertices O =
(1, 1, 0), P = (2, 3, 1), Q = (4, 3, 1), R = (1, 4, 1). Answer: We first see that the
solid is spanned by the vectors ~u = [1, 2, 1]T , ~v = [3, 2, 1]T , and w
~ = [0, 3, 1]T . We
get ~v × w ~ = [−1, −3, 9]T and ~u · (~v × w)
~ = 2. The volume is 2.

3. Problem. A 3D scanner is used to build a 3D model of a face. It detects a


triangle which has its vertices at P = (0, 1, 1), Q = (1, 1, 0) and R = (1, 2, 3). Find
the area of the triangle.

Solution. We have to find the ~ ~


T
√ length of the cross product of P Q and P R which
is [1, −3, 1] . The length is 11.

4. Problem. The scanner detects an other point A = (1, 1, 1). On which side of the
triangle is it located if the cross product of P~Q and P~R is considered the direction
”up”.
Solution. The cross product is ~n = [1, −3, 1]T . We have to see whether the
vector P~A = [1, 0, 0]T points into the direction of ~n or not. To see that, we have
to form the dot product. It is 1 so that indeed, A is ”above” the triangle. Note
that a triangle in space a priori does not have an orientation. We have to tell, what
direction is ”up”. That is the reason that file formats for 3D printing like contain
the data for three points in space as well as a vector, telling the direction.

c Dejen K. 2020 23
1.5. LINES AND PLANES IN SPACE DBU

Exercise 1.4

1. (a) a) Find a unit vector perpendicular to the space diagonal [1, 1, 1]T and the
face diagonal [1, 1, 0]T of the cube.
(b) Find the volume of the parallelepiped for which the base parallelogram is given
by the points P = (5, 2, 2), Q = (3, 1, 2), R = (1, 4, 2), S = (−1, 3, 2) and which
has an edge connecting P with T = (5, 6, 8).
(c) Find the area of the base and use (b) to get the height of the parallelepiped.

~ = ~0. Verify that ~u × ~v = ~v × w


2. (a) Assume ~u + ~v + w ~ × ~u.
~ =w
(b) Find (~u + ~v ) · (~v × w)
~ if ~u, ~v , w
~ are unit vectors which are orthogonal to each
other and ~u × ~v = w. ~

3. To find the equation ax + by + cz = d for the plane which contains the point
P = (1, 2, 3) as well as the line which passes through Q = (3, 4, 4) and R = (1, 1, 2),
we find a vector [a, b, c]T normal to the plane and fix d so that P is in the plane.

1.5 Lines and planes in space


Definition 1.25
A point P = (p, q, r) and a vector ~v = [a, b, c]T define the line
    
 p
 a 

L= q  + t  b  , t ∈ R .
   


r c 

Figure 1.2: Parametric and vector equation of line respectively.

c Dejen K. 2020 24
1.5. LINES AND PLANES IN SPACE DBU

The line consists of all points obtained by adding a multiple of the vector ~v = [a, b, c]T to the
~ = [p, q, r]T . The line contains the point P as well as a copy of ~v attached to P .
vector OP
Every vector contained in the line is necessarily parallel to ~v . We think about the parameter t
as ”time”. At time t = 0, we are at the point P , whereas at time t = 1 we are at OP ~ + ~v .
Definition 1.26
If t is restricted to values in a parameter interval [s, u], then L =
[p, q, r]T + t[a, b, c]T , s ≤ t ≤ u is a line segment which connects ~r(s) with ~r(u).

Example 1.17

To get the line through P = (1, 1, 2) and Q = (2, 4, 6), form the vector ~v = P~Q =
[1, 3, 4]T and get L = [x, y, z]T = [1, 1, 2]T + t[1, 3, 4]T . This can be written also as ~r(t) =
[1 + t, 1 + 3t, 2 + 4t]T . If we write [x, y, z]T = [1, 1, 2]T + t[1, 3, 4]T as a collection of
equations x = 1 + 2t, y = 1 + 3t, z = 2 + 4t and solve the first equation for t:

L = {(x, y, z)|(x − 1)/2 = (y − 1)/3 = (z − 2)/4} .

Theorem 1.10
~ + t~v defined by P = (p, q, r) and vector ~v = [a, b, c]T with nonzero a, b, c
The line ~r = OP
satisfies the symmetric equations
x−p y−q z−r
= = .
a b c
Proof. Each of these expressions is equal to t. These symmetric equations have to be
modified a bit one or two of the numbers a, b, c are zero. If a = 0, replace the first
equation with x = p, if b = 0 replace the second equation with y = q and if c = 0 replace
third equation with z = r. The interpretation is that the line is written as an intersection
of two planes.

Example 1.18

1. Find the symmetric equations for the line through the points P = (0, 1, 1) and
Q = (2, 3, 4), first form the parametric equations [x, y, z]T = [0, 1, 1]6T + t[2, 2, 3]T
or x = 2t, y = 1 + 2t, z = 1 + 3t. Solving each equation for t gives the symmetric
equation x/2 = (y − 1)/2 = (z − 1)/3.

2. Problem: Find the symmetric equation for the z axes. Answer: This is a situa-
tion where a = b = 0 and c = 1. The symmetric equations are simply x = 0, y = 0.
If two of the numbers a, b, c are zero, we have a coordinate plane. If one of the
numbers are zero, then the line is contained in a coordinate plane.

Definition 1.27
A
n point P and two vectors o~v , w
~ define a plane Σ =
~
OP + t~v + sw,
~ where t, s are real numbers .

c Dejen K. 2020 25
1.5. LINES AND PLANES IN SPACE DBU

Example 1.19
n o
An example is σ = [x, y, z]T = [1, 1, 2]T + t[2, 4, 6]T + s[1, 0, −1]T . This is called the
parametric description of a plane.

If a plane contains the two vectors ~v and w, ~ then the vector ~n = ~v ×w~ is orthogonal to both ~v and
~ ~ ~
~ Because also the vector P Q = OQ− OP is perpendicular to ~n, we have (Q−P )·~n = 0. With
w.
Q = (x0 , y0 , z0 ), P = (x, y, z), and ~n = [a, b, c]T , this means ax + by + cz = ax0 + by0 + cz0 = d.
The plane is therefore described by a single equation ax + by + cz = d. We have shown:
Theorem 1.11
The equation for a plane containing ~v and w
~ and a point P is

ax + by + cz = d,

where [a, b, c]T = ~v × w


~ and d is obtained by plugging in P .

Example 1.20
Problem: Find the equation of a plane which contains the three points P =
(−1, −1, 1), Q = (0, 1, 1), R = (1, 1, 3).
Answer: The plane contains the two vectors ~v = [1, 2, 0]T and w ~ = [2, 2, 2]T . We have
~n = [4, −2, −2]T and the equation is 4x − 2y − 2z = d. The constant d is obtained by
plugging in the coordinates of a point to the left. In our case, it is 4x − 2y − 2z = −4.

Definition 1.28
The angle between the two planes ax + by + cz = d and ex + f y + gz = h is defined
as the angle between the two normal vectors ~n = [a, b, c]T and m
~ = [e, f, g]T .

c Dejen K. 2020 26
1.5. LINES AND PLANES IN SPACE DBU

Example 1.21
Problem: Find the angle between the planes x + y = −1 and x + y + √ z = 2. Answer:
T T
~ = [1, 1, 1] . It is arccos(2/ 6).
find the angle between ~n = [1, 1, 0] and m

Finally, lets look at some distance formulas.


1. Sometimes it may be important to find the distance D, between some point P0 and a
plane in the form ax + by + cz + d = 0. The formula for the distance between a plane
Π : ax + by + cz + d = 0 and a point is P (x0 , y0 , z0 ) is as follows:
| ax0 + by0 + cz0 + d |
D= √
a2 + b2 + c2
Proof: Let Q(x1 , y1 , z1 ) be any point on the plane Π and let P0 = (x0 , y0 , z0 ) be our point
~ 0 = (x0 − x1 , y0 − y1 , z0 − z1 ). Recall that this plane can be defined
of interest. Let QP
with a vector that is perpendicular to the plane known as a normal. Let n = (a, b, c) be a
normal to our plane Π. The orthogonal projection of QP ~ 0 onto ~n is equal to the distance
between the plane Π and P0 as illustrated below: Thus it follows that
D = kproj~n QP~ 0k
When simplified and letting d = −ax1 − by1 − cz1
D = kproj~n QP ~ 0k
kQP~ 0 · ~nk
D=
k~nk
| (x0 − x1 , y0 − y1 , z0 − z1 ) · (a, b, c) |
D= √
a2 + b2 + c2
| a(x0 − x1 ) + b(y0 − y1 ) + c(z0 − z1 ) |
D= √
a2 + b2 + c2
| ax0 + by0 + cz0 + d |
D= √
a2 + b 2 + c 2


c Dejen K. 2020 27
1.5. LINES AND PLANES IN SPACE DBU

If P is a point and Σ : ~n · ~x = d is a plane containing a point Q, then

|P~Q · ~n|
d(P, Σ) =
|~n|

is the distance between P and the plane.


Proof: use the angle formula in the denominator. For example, to find the distance from
P = (7, 1, 4) to σ : 2x + 4y + 5z = 9, we find first a a point Q = (0, 1, 1) on the plane.
Then compute
|[−7, 0, −3]T · [2, 4, 5]T | 29
d(P, Σ) = T
=√ .
|[2, 4, 5] | 45
2. The distance between a point and a line, is defined as the shortest distance between a fixed
point and any point on the line. It is the length of the line segment that is perpendicular
to the line and passes through the point.

Let Q be the point with coordinates (x0 , y0 ) and let the given line have equation ax +
by + c = 0. Also, let p = (x1 , y1 ) be any point on this line and n the vector (a, b)
starting at point p. The vector n is perpendicular to the line, and the distance d from
−→
point Q to the line is equal to the length of the orthogonal projection of P Q on n. The

c Dejen K. 2020 28
1.5. LINES AND PLANES IN SPACE DBU

−→
|P Q · n| −→
length of this projection is given by: d = . Now, P Q = (x0 − x1 , y0 − y1 ), , so
knk
−→ √ |a(x0 − x1 ) + b(y0 − y1 )|
P Q · n = a(x0 − x1 ) + b(y0 − y1 ) and knk = a2 + b2 , thus d = √ .
a2 + b 2
Since P is a point on the line, c = −ax1 − by1 , and so,
|ax0 + by0 + c|
d= √ .
a2 + b 2

If P is a point in space and L is the line ~r(t) = Q + t~u, then

|(P~Q) × ~u|
d(P, L) =
|~u|
is the distance between P and the line L.
Proof: the area divided by base length is height of parallelogram.
For example, to compute the distance from P = (2, 3, 1) to the line ~r(t) = (1, 1, 2) +
t(5, 0, 1), compute

|[−1, −2, 1]T × [5, 0, 1]T | |[−2, 6, 10]T | 140
d(P, L) = T
= √ = √
[5, 0, 1] 26 26

3. If L is the line ~r(t) = Q + t~u and M is the line ~s(t) = P + t~v , then

|(P~Q) · (~u × ~v )|
d(L, M ) =
|~u × ~v |
is the distance between the two lines L and M .
Proof: the distance is the length of the vector projection of P~Q onto ~u × ~v which is
normal to both lines.
For example, to compute the distance between ~r(t) = (2, 1, 4) + t(−1, 1, 0) and M is
the line ~s(t) = (−1, 0, 2) + t(5, 1, 2) form the cross product of [−1, 1, 0]T and [5, 1, 2]T is
[2, 2, −6]T . The distance between these two lines is
|(3, 1, 2) · (2, 2, −6)| 4
d(L, M ) = T
=√ .
|[2, 2, −6] | 44

4. To get the distance between two planes ~n · ~x = d and ~n · ~x = e, then their distance is
|e − d|
d(Σ, Π) =
|~n|

c Dejen K. 2020 29
1.5. LINES AND PLANES IN SPACE DBU

Non-parallel planes have distance 0. Proof: use the distance formula between point and
plane. For example, 5x + 4y + 3z = 8 and 10x + 8y + 6z = 2 have the distance

|8 − 1| 7
T
=√ .
|[5, 4, 3] | 50

Exercise 1.5

1. Given the three points P = (7, 4, 5) and Q = (1, 3, 9) and R = (4, 2, 10). find the
parametric and symmetric equation for the line perpendicular to the triangle P QR
passing through its center of mass (P + Q + R)/3.

2. Find a parametric equation for the line through the point P = (3, 1, 2) that is
perpendicular to the line L : x = 1 + 4t, y = 1 − 4t, z = 8t and intersects this line
in a point Q.

3. (a) Find the equation of the plane containing the three points P = (1, 1, 1), Q =
(4, 6, 3), R = (5, 5, 0).
(b) Find the area of the triangle P QR.

4. (a) Compute a suitable volume to determine whether A = (2, 2, 3), B =


(4, 0, 7), C = (6, 3, 1) and D = (2, −3, 11) are in the same plane.
(b) Find the distance between the line L through A, B and the line M through
C, D.

5. (a) Find an equation of the plane containing the line of intersection of the planes
x − z = 1 and y + z = 3 which is perpendicular to the plane x + y − 2z = 1.
(b) Find the distance of the plane found in (a) to the origin (0, 0, 0).

6. Given three spheres of radius 9 centered at A = (1, 2, 0), B = (4, 5, 0), C = (1, 3, 2).
Find a plane ax + by + cz = d which touches all of three spheres from the same
side.

c Dejen K. 2020 30
1.6. VECTOR SPACE AND SUBSPACE DBU

1.6 Vector space and Subspace


Definition 1.29
A vector space (over R ) consists of a set V along with two operations + and · subject
to these conditions.

~ ∈ V , their vector sum (~v + w)


1. For any ~v , w ~ is an element of V .

~ ∈ V , then ~v + w
2. If ~v , w ~ =w
~ + ~v .

~ ∈ V, (~v + w)
3. For any ~u, ~v , w ~ + ~u = ~v + (w
~ + ~u) .

4. There is a zero vector 0 ∈ V such that ~v + 0 = ~v for all ~v ∈ V .

5. Each ~v ∈ V has an additive inverse w


~ ∈ V such that w
~ + ~v = 0.

6. If r is a scalar, that is, a member of R and ~v ∈ V then the scalar multiple r·~v is in
V.

7. If r, s ∈ R and ~v ∈ V then (r + s) · ~v = r · ~v + s · ~v .

8. If r ∈ R and ~v , w
~ ∈ V , then r · (~v + w)
~ = r · ~v + r · w.
~

9. If r, s ∈ R and ~v ∈ V , then (rs) · ~v = r · (s · ~v ).

10. For any ~v ∈ V, 1 · ~v = ~v .

c Dejen K. 2020 31
1.6. VECTOR SPACE AND SUBSPACE DBU

Example 1.22

Show that the set R2 is a vector space if the operations ” + ” and ” · ” have their usual meaning.
     
x1 y1 x1 + y1
+ =
x2 y2 x2 + y2
   
x1 rx1
&r · =
x2 rx2

Solution:To prove R2 is a vector space we shall check all of the conditions.


There are five conditions in item 1.
For 1, closure of addition, note that for any v1 , v2 , w1 , w2 ∈ R the result of the sum

     
v1 w1 v + w1
+ = 1
v2 w2 v2 + w2

is a column array with two real entries, and so is in R2 .


For 2, that addition of vectors commutes, take all entries to be real numbers and compute
     
v1 w1 v1 + w 1
+ =
v2 w2 v2 + w 2
 
w 1 + v1
=
w 2 + v2
   
w1 v
= + 1
w2 v2

(the second equality follows from the fact that the components of the vectors are real numbers, and the
addition of real numbers is commutative).
Condition 3, associativity of vector addition, is similar.
       
v1 w1 u1 (v1 + w1 ) + u1
{ + }+ =
v2 w2 u2 (v2 + w2 ) + u2 
v + (w1 + u1 )
= 1
v
 2 + (w2+ u2)  
v1 w1 u1
= +{ + }
v2 w2 u2

For the fourth condition we must produce a zero element the vector of zeroes is it.
     
v1 0 v
+ = 1
v2 0 v2

For 5, to produce an additive inverse, note that for any v1 , v2 ∈ R we have


     
−v1 v 0
+ 1 =
−v2 v2 0

so the first vector is the desired additive inverse of the second.


To check the next five conditions having to do with scalar multiplication are just as routine.
For 6, closure under scalar multiplication, where

r, v1 , v2 ∈ R,
   
v rv1
r· 1 =
v2 rv2

is a column array with two real entries, and so is in R2 .

c Dejen K. 2020 32
1.6. VECTOR SPACE AND SUBSPACE DBU

Example 1.23

Next, this checks 7.


   
v1 (r + s)v1
(r + s) · =
v2 (r + s)v2
 
rv1 + sv1
=
rv2 + sv2
   
v v
=r· 1 +s· 1
v2 v2

For 8, that scalar multiplication distributes from the left over vector addition, we have this.

     
v w1 r(v1 + w1 )
r·( 1 + )=
v2 w2 r(v2 + w2 )
 
rv1 + rw1
=
rv2 + rw2
   
v w1
=r· 1 +r·
v2 w2

The ninth
   
v1 (rs)v1
(rs) · =
v2 (rs)v2
 
r(sv1 )
=
r(sv2 )
 
v
= r · (s · 1 )
v2

and tenth conditions are also straightforward.

     
v 1v1 v
1· 1 = = 1
v2 1v2 v2

In a similar way, each Rn where n ∈ N is a vector


  space with the usual operations of vector addition
0
0
0} is a vector space under the operations
and scalar multiplication. The singleton set { 

0
     
0 0 0
0 0 0
 + = 
0 0 0
0 0 0
   
0 0
0 0
&r · 
0 = 0
  

0 0

c Dejen K. 2020 33
1.6. VECTOR SPACE AND SUBSPACE DBU

Exercise 1.6

1. Show that P3 = {a0 + a1 x + a2 x2 + a3 x3 a0 , . . . , a3 ∈ R}, the set of polynomials of


degree three or less is a vector space under the operations

(a0 +a1 x+a2 x2 +a3 x3 )+(b0 +b1 x+b2 x2 +b3 x3 ) = (a0 +b0 )+(a1 +b1 )x+(a2 +b2 )x2 +(a3 +b3 )x3

and
r · (a0 + a1 x + a2 x2 + a3 x3 ) = (ra0 ) + (ra1 )x + (ra2 )x2 + (ra3 )x3

2. The set M2×2 of 2×2 matrices with real number entries is a vector space under the
natural
! entry-by-entry
! operations. ! ! !
a b w x a+w b+x a b ra rb
+ = and r · =
c d y z c+y d+z c d rc rd

3. The set of polynomials with real coefficients {a0 + a1 x + · · · + an xn n ∈


N and a0 , . . . , an ∈ R}
makes a vector space when given the natural ” + ”

(a0 +a1 x+· · ·+an xn )+(b0 +b1 x+· · ·+bn xn ) = (a0 +b0 )+(a1 +b1 )x+· · ·+(an +bn )xn

and
r · (a0 + a1 x + . . . an xn ) = (ra0 ) + (ra1 )x + . . . (ran )xn

Subspace
A subspace is a vector space that is contained within another vector space. So every subspace
is a vector space in its own right, but it is also defined relative to some other (larger) vector
space. We will discover shortly that we are already familiar with a wide variety of subspaces
from previous sections.
Definition 1.30
Suppose that V and W are two vector spaces that have identical definitions of vector
addition and scalar multiplication, and that W is a subset of V , W v V . Then W is a
subspace of V .

Proposition:- A subset S of a vector space V is a subspace of V if and only if S is nonempty


and closed under linear operations, i.e., x, y ∈ S ⇒ x + y ∈ S, and x ∈ S ⇒ rx ∈ S for
all r ∈ R.

c Dejen K. 2020 34
1.7. LINEAR DEPENDENCE, INDEPENDENCE AND BASIS DBU

Example 1.24
 
x
The set P = {y  x + y + z = 0} is a subspace of R3 if ” + ” and ” · ” are interpreted
 

z
         
x1 x2 x1 + x2 x rx
as  y +
  
 1  2 y = 
y
 1 + y 
2 and r · y
 
  = ry 
 

z1 z2 z1 + z2 z rz
Solution:- Since R is a vector space and also P is a subset of R3 then by the above
3

preposition we prove that P is closed under addition and scalar multiplication , take two
elements
  ofP 
x1 x2
 
y
 1 and  y2  (membership in P means that x1 + y1 + z1 = 0 and x2 + y2 + z2 = 0),
 

z1 z2
 
x1 + x2
and observe that their sum   y1 + y2  is also in P since its entries add (x1 + x2 ) + (y1 +

z1 + z2
y2 ) + (z1 + z2 ) = (x1 + y1 + z1 ) + (x2 + y2 + z2 ) = 0 + 0 = 0. To show that P is closed
under
  scalar multiplication, start with a vector from P
x
y  (so that x + y + z = 0 ) and then for r ∈ R observe that the scalar multiple
 

z
   
x rx
r · y  = ry  satisfies that rx + ry + rz = r(x + y + z) = r.0 = 0. Thus the two closure
   

z rz
conditions are satisfied. There fore P is a subspace of R3 .

Exercise: Show that a nonempty subset S of a real vector space is a subspace if and only
if it is closed under linear combinations of pairs of vectors: whenever c1 , c2 ∈ R and ~v1 , ~v2 ∈ S
then the combination c1~v1 + c2~v2 is in S.

1.7 Linear dependence, independence and Basis


A set of vectors is linearly independent if no vector in the set V is a scalar multiple of another
vector or a linear combination of other vectors in the set;
conversely, a set of vectors is linearly dependent if any vector in the set V is a scalar multiple
of another vector in the set or a linear combination of other vectors in the set.

Definition 1.31
Suppose that V is a vector space. Given n vectors u1 , u2 , u3 , · · · un and n scalars
α1 , α2 , α3 · · · , αn , their linear combination is the vector α1 u1 + α2 u2 + α3 u3 + · · · + αn un .

c Dejen K. 2020 35
1.7. LINEAR DEPENDENCE, INDEPENDENCE AND BASIS DBU

Definition 1.32
If S = {v1 , v2 , v3 , · · · , vn } is a non-empty set of vectors, then the vector equation
k1 v1 + k2 v2 + k3 v3 + · · · + kn vn = 0 has at least one trivial solution, namely k1 = k2 =
k3 = · · · = kn = 0. If this solution is the only solution of the set S then the set S is called
linearly independence set.
if there are other solutions then the set S is called linearly dependent.

Example 1.25

Let v1 = (1, 0, 1), v2 = (−1, 1, 0) and v3 = (1, 2, 3). Express v3 as a linear combination
of v1 and v2 .
Solution: We must find scalars c1 and c2 so that v3 = c1 v1 + c2 v2 Using our knowledge
of scalar multiplication and addition of vectors, we set

(1, 2, 3) = c1 (1, 0, 1) + c2 (−1, 1, 0)


(1, 2, 3) = (c1 , 0, c1 ) + (−c2 , c2 , 0)
(1, 2, 3) = (c1 − c2 , c2 , c1 )

Equating corresponding components we have :


1 = c1 − c2 , 2 = c2 & 3 = c1
Since c1 = 3 and c2 = 2 does satisfy c1 − c2 = 1, we see that this is the solution. Hence
(1, 2, 3) = 3(1, 0, 1) + 2(−1, 1, 0). so we have found the required linear combination to be
v3 = 3v1 + 2v2 .

Example 1.26

Determine whether {v1 , v2 , v3 } form a linearly independent set or not, where v1 =


(1, 1, 2, 1), v2 = (0, 2, 1, 1), and v3 = (3, 1, 2, 0)
Solution: Using the second definition, we wish to know whether c1 v1 + c2 v2 + c3 v3 = 0
has only the trivial solution.
Suppose that c1 v1 + c2 v2 + c3 v3 = 0.
Then

c1 (1, 1, 2, 1) + c2 (0, 2, 1, 1) + c3 (3, 1, 2, 0) = 0


(c1 , c1 , 2c1 , c1 ) + (0, 2c2 , c2 , c2 ) + (3c3 , c3 , 2c3 , 0) = 0
(c1 + 3c3 , c1 + 2c2 + c3 , 2c1 + c2 + 2c + 3, c1 + c2 ) = 0
c1 + 3c3 = 0
c1 + 2c2 + c3 = 0
2c1 + c2 + 2c + 3 = 0
c1 + c2 = 0

has the unique solution c1 = c2 = c3 = 0.


There fore we see that the set {v1 , v2 , v3 } is linearly independent.

c Dejen K. 2020 36
1.7. LINEAR DEPENDENCE, INDEPENDENCE AND BASIS DBU

Theorem 1.12
Suppose that S = {u1 , u2 , u3 , · · · , un } is a set of vectors. Then S is a linearly dependent
set if and only if there is an index t 1 6 t 6 n such that ut is a linear combination of the
vectors u1 , u2 , u3 , · · · ut−1 , ut+1 , · · · , un .

Definition 1.33
Let V denote a vector space and S = {u1 , u2 , u3 , · · · , un } a subset of V. S is called a
basis for V if the following is true:

1. S spans V.

2. S is linearly independent.

Definition 1.34
Let V be a vector space and let S = {v1 , v2 , · · · , vn } be a subset of V. We say that S
spans V if every vector v in V can be written as a linear combination of vectors in S.
v = c1 v1 + c2 v2 + · · · + cn vn

Example 1.27

Show that the set S = {(0, 1, 1), (1, 0, 1), (1, 1, 0)} spans R3 and write the vector (2, 4, 8)
as a linear combination of vectors in S.
Solution:- A vector in R3 has the form v = (x, y, z) Hence we need to show that every
such v can be written as

(x, y, z) = c1 (0, 1, 1) + c2 (1, 0, 1) + c3 (1, 1, 0)


= (c2 + c3 , c1 + c3 , c1 + c2 )
x = c2 + c3
y = c1 + c3
z = c1 + c2
c3 =x − c2 ,
&c1 =z − c2
y = z − c2 + x − c2
= z + x − 2c2
c2 = (x − y + z)/2,
c1 = (z − x + y)/2
c3 = (x + y − z)/2

(z − x + y) (x − y + z) (x + y − z)
∴ for every (x, y, z) = (0, 1, 1) + (1, 0, 1) + (1, 1, 0).
2 2 2
Hence S = {(0, 1, 1), (1, 0, 1), (1, 1, 0)} spans R3
And We have (2, 4, 8) = 5(0, 1, 1) + 3(1, 0, 1) + (−1)(1, 1, 0)

c Dejen K. 2020 37
1.7. LINEAR DEPENDENCE, INDEPENDENCE AND BASIS DBU

Example 1.28

Prove that S = {1, x, x2 } is a basis for the set of polynomials of degree less than or
equal to 2.
Solution: We need to prove that S spans P2 and is linearly independent.
S spans P2 . We already did this in the section on spanning sets. A typical polynomial
of degree less than or equal to 2 is ax2 + bx + c

S is linearly independent. Here, we need to show that the only solution to a+bx+cx2 = 0
(where 0 is the zero polynomial) is a = b = c = 0.
From algebra, we remember that two polynomials are equal if and only if their corre-
sponding coefficients are equal. The zero polynomial has all its coefficients equal to zero.
So, a(1) + bx + cx2 = 0 if and only if a = 0, b = 0, c = 0. Which proves that S is
linearly independent.

Theorem 1.13
Let V denote a vector space and S = {u1 , u2 , u3 , · · · , un } a basis of V. Every vector in
V can be written in a unique way as a linear combination of vectors in S

c Dejen K. 2020 38
1.7. LINEAR DEPENDENCE, INDEPENDENCE AND BASIS DBU

Exercise 1.7

1. A kite is pulled with a force F~ = h2, 1, 4i. It has velocity ~v = h1, −1, 1i. The dot product of F~
with ~v is called power.
(a) Find the angle between the force and the velocity.
(b) Find the vector projection of the force onto the velocity vector.
2. Light shines long the vector ~a = ha1 , a2 , a3 i and reflects at the three coordinate planes where the
angle of incidence equals the angle of reflection. Verify that the reflected ray is ~a.
Hint. Reflect first at the xy-plane and find the components of the ray after reflection. You can
assume that in that case, the reflected vector is in the plane spanned by ~k = h0, 0, 1i and ~a.
3. Given the vectors ~a = h1, 2, 1i, ~b = h1, −1, 1i, ~c = h0, 1, 1i, d = h2, 3, 4i, compute all possible dot
products and determine which pairs are perpendicular.
4. (a) Find the angle between a diagonal of a cube and the diagonal in one of its faces.
(b) The hypercube or tesseract has vertices (±1, ±1, ±1, ±1). Find the angle between the hyper
diagonal connecting (1, 1, 1, 1) with (−1, −1, −1, −1) and the space diagonal connecting
(1, 1, 1, 1) with (−1, −1, −1, −1)
5. (a) Verify that if ~a. ~b are nonzero, then ~c =| ~a | ~b+ | ~b | ~a bisects the angle between ~a. ~b if ~c is
not zero.
(b) Verify the parallelogram law

| ~a + ~a |2 + | ~a − ~a |2 = 2 | ~a |2 +2 | ~b |2 .

6. (a) Find a nonzero vector orthogonal to the plane through the points P = (−2, 3, 1), Q =
(1, 5, 2), R = (4, 3, −1) and containing P .
(b) Find the equation of this plane.
(c) Find the area of the triangle P QR.
7. (a) Parametrize a line perpendicular to the plane containing A = (1, 1, .1), B = (2, 3, 4) and
C = (4, 5, 6) and passing through A.
(b) Find the equation ax + by + cz = d of the plane through A, B, C.
8. (a) Use volume to determine whether A = (1, 1, 2), B = (3, 1, 6), C = (5, 2, 0) and D = (1, 4, 12)
are in the same plane.
(b) Find the distance between the line L through A, B and the line M through C, D.
(c) How come that whenever A, B, C, D are not in the same plane, then the distance between
L and M is positive?
9. (a) Find an equation of the plane containing the line of intersection of the planes x − z = 1 and
y + z = 3 which is perpendicular to the plane x + y − 2z = 1.
(b) Find the distance of the plane found in a) to the origin (0, 0, 0).
10. (a) Parametrize the line L through P = (2, 1, 2) that intersects the line x = 1 + t, y = 1 − t, z =
2t perpendicularly.
(b) What is the distance from this line L to the origin (0, 0, 0)?

c Dejen K. 2020 39
Debre Brehan University
Applied Mathematics One(Math 1041)
Lecture Note
”Mathematics is the most beautiful and most powerful creation of
the human spirit.”
Stefan Banach

Dejen Ketema
Department of Mathematics
[email protected]

March, 2020
Contents

1 Matrices and Determinants 2


1.1 Definition of matrix and basic operations . . . . . . . . . . . . . . . . . . . . . . 2
1.1.1 Types of Matrices . . . . . . . . . . . . . . . . . . . . . . . . . . . . . . . 4
1.1.2 Operations on Matrices . . . . . . . . . . . . . . . . . . . . . . . . . . . 6
1.1.3 Multiplication of Matrices by Scalar . . . . . . . . . . . . . . . . . . . . 7
1.2 Product and Transpose of a matrix . . . . . . . . . . . . . . . . . . . . . . . . . 8
1.2.1 The Transpose of a Matrix . . . . . . . . . . . . . . . . . . . . . . . . . 9
1.2.2 Trace of a Matrix . . . . . . . . . . . . . . . . . . . . . . . . . . . . . . . 9
1.3 Elementary Row Operations and its properties . . . . . . . . . . . . . . . . . . . 10
1.4 Inverse of a matrix and its properties . . . . . . . . . . . . . . . . . . . . . . . . 13
1.5 Determinant of a matrix and its properties . . . . . . . . . . . . . . . . . . . . . 16
1.6 Solving system of linear equations . . . . . . . . . . . . . . . . . . . . . . . . . . 18
1.6.1 Cramer’s rule . . . . . . . . . . . . . . . . . . . . . . . . . . . . . . . . . 21
1.6.2 Gaussian elimination method . . . . . . . . . . . . . . . . . . . . . . . . 23
1.6.3 Inverse matrix method . . . . . . . . . . . . . . . . . . . . . . . . . . . . 24
1.7 Eigenvalues and Eigenvectors . . . . . . . . . . . . . . . . . . . . . . . . . . . . 26
1.7.1 Diagonalization . . . . . . . . . . . . . . . . . . . . . . . . . . . . . . . . 28

1
Chapter 1

Matrices and Determinants

Objective:

• By the end of this chapter, students are expected to:

– Define and identify different types of matrices


– Understand the arithmetic operations on matrices
– Reduce the given matrix to row reduced echelon form using elementary row opera-
tions
– Find the inverse of some matrices using elementary row operations
– Define system of linear equations in terms of matrices
– Apply Gaussian elimination method, Gaussian Jordan method, and matrix inversion
method to solve the given system of linear equations
– Define and compute eigenvalue and eigenvectors

1.1 Definition of matrix and basic operations


Definition 1.1
Matrix is a rectangular array or arrangement of numbers of the form
 
a11 a12 a13 ··· a1n
 a21 a22 a23 ··· a2n 
 
 
a
A= 31 a32 a33 ··· a3n 

 . .. .. ... .. 
 .. . . . 
 
am1 am2 am3 · · · amn

is called a matrix of size m by n (written as m × n).

Each number aij is called an element or entry of the matrix and it is an element appearing in
the ith row and j th column of a matrix. Elements in the horizontal line are said to form rows,
and elements in the vertical lines are said hto form columns. Herei we say A has m rows and n
columns. The ith row of matrix A is Ri = ai1 ai2 ai3 · · · ain 1 ≤ i ≤ m

2
The j th column of the matrix A is
 
a1j
 a2j 
 
 
a 
cj =  3j  1 ≤ j ≤ n.
 . 
 .. 
 
amj

The order of a matrix denotes the number of rows and columns in the matrix. Thus, a matrix
of order m × n has m rows and n columns. We often write the matrix A as A = [aij ]m×n , 1 ≤
i ≤ m, 1 ≤ j ≤ n, i-denotes the row and j- denotes the column.
For example, in the matrix  
5 9 6 8
A = 3 2 3 1
 

1 0 4 7
, there are 3 rows and 4 columns. Therefore, matrix A can be called a matrix of order or size
3 × 4.
The rows are h i h i h i
5 9 6 8 , 3 2 3 1 & 1 0 4 7
, and the columns        
5 9 6 8
3 , 2 , 3 & 1 .
       

1 0 4 7
Here a11 = 5, a12 = 9, a13 = 6, a14 = 8, a21 = 3, a22 = 2, a23 = 3, a24 = 1, etc. and A has 12
elements.

Definition 1.2
Two matrices A and B are said to be equal, written A = B, if they are of the same order
and if all corresponding entries are equal i.e. aij = bij .

For example, " # " #


5 1 0 2+3 1 0
=
2 3 4 2 3 2×2
but " #
2 h i
6= 2 9
9
. Why?
Example 1.1
Given the matrix equation " # " #
x+y 8 3 8
= .
x−y 6 1 6
Find x and y.
Solution: By the definition of equality of matrices,
x + y = 3 and x − y = 1 solving this system of equations gives x = 2 and y = 1.

3
Exercise 1.1

1. Find the values of x, y, z and w which satisfy the matrix equation


" # " # " # " #
x − y 2x + z −1 5 x + 3 2y + x 0 −7
(a) = (b) =
2x − y 3z + w 0 13 z − 1 4w + 6 3 2w

1.1.1 Types of Matrices


1. A matrix having exactly one row is called a row matrix.

Example 1.2
   
1 0 4 7 , 1 5
, are row matrices. A row matrix is often referred to as a row vector.

2. A matrix having exactly one column is called a column matrix

Example 1.3
 
4
 3 
 

−6
is a column matrix.

3. A zero matrix or null matrix is a matrix in which all of its elements are zero.
Example 1.4
The matrices,  
" # 0 0
0 0
& 0 0
 
0 0
0 0
are the zero matrices

4. A matrix, in which the numbers of rows and the number of columns are equal, that is,
an n × n matrix, is called a square matrix of order n.

Example 1.5
 
" # 1 2 8
3 1
, & 4 6 0
 
4 0
1 3 5
are square matrices.

If A = (aij )n×n is a square matrix the elements aii ’s are called the diagonal elements.
The main diagonal or simply diagonal of A consists of the elements a11 , a22 , a33 , · · · , ann .

4
5. A square matrix in which all the non-diagonal elements are zero is called a diagonal matrix.

Example 1.6
 
" # 4 0 0
3 0
, & 0 5 0
 
0 2
0 0 9
are diagonal matrices.

Note that the diagonal elements in a diagonal matrix may also be zero.

Example 1.7
" # " #
0 0 0 0
,&
0 2 0 0
are also diagonal matrix.

6. A diagonal matrix whose diagonal elements are equal is called a scalar matrix

Example 1.8
 
" # 4 0 0 " #
3 0 0 0
, & 0 4 0 &
 
0 3 0 0
0 0 4
are scalar matrices

7. Diagonal matrix of order n in which every diagonal element is unity (one) is called the
identity matrix or unit matrix of order n . The identity matrix of order n is denoted by
In .
Example 1.9
" #
1 0
0 1
is an identity matrix of order two.

8. A square matrix having only zeros below its diagonal is called upper triangular matrix.
A square matrix having only zeros above its diagonal is called lower triangular matrix. A
matrix that is either upper triangular or lower triangular is called triangular matrix.

5
Example 1.10
 
1 0 0 " #
3 0
2 0 0 , &
 
6 4
0 4 3
are lower triangular matrices and
 
1 2 0 " #
3 7
0 0 0 , &
 
0 4
0 0 3

are upper triangular matrices.

Definition 1.3
A matrix obtained by deleting one or more rows and/or columns of A is called sub matrix
of A.

Example 1.11
   
4 6 1 " # " # 4 6
8 2 4 6
If A = 3 8 2, then , and 3 8 are some of the sub matrices of A.
   
0 3 3 8
2 0 3 2 0

1.1.2 Operations on Matrices


Addition and Subtraction of matrices
Definition 1.4
If A and B are matrices of the same order, then sum of A and B , denoted by A + B,
is the new matrix of the same order obtained by adding the corresponding elements of A
and B. Similarly, the difference of A and B , denoted by A − B, is the matrix obtained
by subtracting the corresponding elements ofA and B.

Example 1.12
   
1 2 8 2 0 3
1. IfA = 4 6 0 and B = 3 8 2
  
, then we have that
1 3 5 4 6 1
       
1 2 8 2 0 3 1+2 2+0 8+3 3 2 11
(a) A + B = 4 6 0 + 3 8 2 = 4 + 3 6 + 8 0 + 2 = 7 14 2 
       

1 3 5 4 6 1 1+4 3+6 5+1 5 9 6


       
1 2 8 2 0 3 1−2 2−0 8−3 −1 2 5
(b) A − B = 4 6 0 − 3 8 2 = 4 − 3 6 − 8 0 − 2 =  1 −2 −2.
       

1 3 5 4 6 1 1−4 3−6 5−1 −3 −3 4

6
1.1.3 Multiplication of Matrices by Scalar
Let A be any matrix and α be a scalar (real number), then αA is the matrix obtained from A
multiplying each element of A by α. This operation is called scalar multiplication.
In particular, −A is the matrix obtained from A by multiplying each element of A by −1 and
is called the additive inverse of A.
Example 1.13
 
1 2 6
If A = 5 0 4 , then
 

3 1 2
     
1 2 6 3(1) 3(2) 3(6) 3 6 18
5 0 4 = 3(5) 3(0) 3(4) = 15 0 12
3A = 3      

3 1 2 3(3) 3(1) 3(2) 9 3 6

Properties on Matrix addition and Scalar Multiplication


Let A, B and C be m × n matrices and 0 be a zero matrix of size m × n and α, β be scalars.
Then

1. A + B = B + A (Commutative law for addition)

2. (A + B) + C = A + (B + C) (Associative law for addition)

3. A + 0 = 0 + A = A ( 0 is called Additive identity )

4. For each matrix A , there exists a unique m × n matrix −A such that A + (−A) = 0 =
−A + A

5. α(A + B) = αA + αB

6. (αβ)A = α(βA) = β(αA)

7. (α + β)A = αA + βA

From the above properties, the set of all matrices having the same order forms a vector space
with the operations addition and scalar multiplication.

7
1.2 Product and Transpose of a matrix
Definition 1.5: ( Matrix Product)

Let A be an m × r matrix and B be an r × n. The (ij)th entry of C = AB is the dot


product of the ith row vector of A and the j th column vector of B:
 
b1j
b 
 
h i  2j 
cij = ai1 ai2 ai3 · · · b3j 
air ·  
 . 
 .. 
 
brj
= ai1 b1j + ai2 b2j + ai3 b3j + · · · + air brj
= Σrk=1 aik bkj

The product C has order m × n.

Example 1.14
   
1 1 3 2 5
1. If A = 1 0 5

, B = 2 3

 then
3 2 1 4 1
 
1(2) + 1(2) + 3(4) 1(5) + 1(3) + 3(1)
AB = 1(2) + 0(2) + 5(4) 1(5) + 0(3) + 5(1)
 

3(2) + 2(2) + 1(4) 3(5) + 2(3) + 1(1)


 
2 + 2 + 12 5 + 3 + 3
= 2 + 0 + 20 5 + 0 + 5 
 

6 + 4 + 4 15 + 6 + 1
 
14 11
= 22 10
 

14 22
" # " # " # " #
2 3 0 3 6 9 21 0
2. A = , B= , then AB = and BA = . So, AB 6= BA
7 0 2 1 0 21 11 6

Note: In general matrix multiplication is not commutative.

Properties of Matrix Multiplication


Let A, B and C be three matrices of the appropriate sizes. Let α be a scalar. Then

1. A(BC) = (AB)C.

2. A(B + C) = AB + AC and (A + B)C = AC + BC.

3. α(AB) = (αA)B = A(αB)

8
1.2.1 The Transpose of a Matrix
Definition 1.6
A matrix obtained from a given matrix A by interchanging the rows and columns is called
the transpose of A and it is denoted by At .That is, If A = (aij )m×n then At = (aji )n×m

Example 1.15
 
2 5 " #
t 2 −2 4
1. If A = −2 3, then A =
 
5 3 1
4 1
 
4 h i
2. If B =  3 , then B t = 4 3 −6
 

−6

Let A = (aij )n×n be a square matrix. Then A is said to be


i) Symmetric matrix if At = A
ii) Skew symmetric if At = −A
Example 1.16
   
2 3 4 2 3 4
t
1. A = 3 0 −2 , A = 3 0 −2
  
,
4 −2 1 4 −2 1
=⇒ At = A, therefore, A is symmetric.
" # " #
0 −1 0 1
2. A = , At = , therefore, A is skew-symmetric.
1 0 −1 0

Properties of Transpose of Matrix


Let A and B be matrices such that addition and multiplication is defined. Then
1. (At )t = A
2. (A + B)t = At + B t And (AB)t = B t At
3. (αA)t = αAt , α − is a scalar
1 1
(A + At ) + (A − At )
4. A = |2 {z } |2 {z }
symmetric skew

1.2.2 Trace of a Matrix


Definition 1.7
let A = (aij )n×n , be a square matrix of order n .Then trace of A is defined to be the sum
of the diagonal elements of A. That is trace(A) = ni=1 aii .
P

Notation: The trace of a matrix A is also commonly denoted as trace(A) or tr(A).

9
Properties of trace of a matrix
If A and B are square matrices, then

• trace(A + B) = trace(A) + trace(B)

• trace(A) + trace(At )

• trace(cA) = c(trace(A))

• trace(AB) = trace(BA)

Example 1.17
 
15 6 7
Find the trace of A =  2 −4 2


3 2 6
Solution: tr(A) = 3i=1 aii = 15 + −4 + 6 = 17
P

1.3 Elementary Row Operations and its properties


Definition 1.8: (Elementary row operation)

Given any matrix Aof order m × n. Any one of the following operations on the matrix is
called elementary row operation.

1. Interchanging any two rows of A Ri ⇔ Rj (Interchange the ith and j th row)

2. Multiplying a row of A by a nonzero constant k Ri =⇒ kRi (Multiply the ith row by


scalar k)

3. Adding a multiple of one row of A to another row of A . Rj =⇒ Rj + kRi (add k


times ith row to j th row).

10
Example 1.18
 
0 0 1 2
1. Give a matrix A = 2 3 0 −2


3 3 6 −9

(a) Interchange

rows 1 and 3 of A
3 3 6 −9
=⇒ 2 3 0 −2
 

0 0 1 2
(b) Multiply

the third row

of A by 1/3
0 0 1 2
=⇒ 2 3 0 −2
 

1 1 2 −3
(c) Multiply

the second row

of A by −2, then add to the third row of A
0 0 1 2
=⇒  2

3 0 −2 
−1 −3 6 −5

Definition 1.9
Two matrices A and B are called row equivalent or simply (equivalent matrices) if one
matrix can be obtained from the other matrix by applying finite number of elementary
operations. In this case we write A ∼ B.

Example 1.19
As we observe from the above example,
   
3 3 6 −9 3 3 6 −9
2 3 0 −2 ∼ 2/3 1 0 −2/3
   

0 0 1 2 0 0 1 2

and    
3 3 6 −9 0 0 1 2
2 3 0 −2 ∼  2 3 0 −2
  

0 0 1 2 −1 −3 6 −5

Definition 1.10: (Matrix in reduced row echelon form):

A matrix in reduced row echelon form has the following properties:

1. All rows consisting entirely of 0 are at the bottom of the matrix.

2. For each nonzero row, the first entry is 1. The first entry is called a leading 1.

3. For two successive non zero rows, the leading 1 in the higher row appears farther
to the left than the leading 1 in the lower row.

4. If a column contains a leading 1, then all other entries in that column are 0.

Note: A matrix is in row echelon form as the matrix has the first 3 properties.

11
Example 1.20
   
1 2 0 0 2 1 0 0 3 0
0 0 1 0 1 0 0 1 0 0
   
   
A= 0 0 0 1 0  and B = 0 0 0
  0 1
0 0 0 0 0 0 0 0 0 0
  

0 0 0 0 0 0 0 0 0 0
are the matrices in reduced row echelon form. Where as the matrix.

1 0 3 4
 
0
 1 −2 5

0 0 1 2
 

0 0 0 0

is not in reduced row echelon form but it is row echelon form since the matrix has the first
3 properties and all the other entries above the leading 1 in the third column are not 0. The
matrix
1 0 3 4
 
0
 1 −2 5

0 1 2 2
 

0 0 0 0

is not in row echelon form (also not in reduced row echelon form) since the leading 1 in the
second row is not in the left of the leading 1 in the third row and all the other entries above
the leading 1 in the third column are not 0.

Definition 1.11: (Rank of a matrix)

The rank of a matrix A , denoted by rank(A), is the number of nonzero rows remaining
after it has been changed into row echelon or reduced row echelon form.

Remark: If A is zero matrix then rank(A) is 0.

12
Example 1.21
Determine the rank of the following matrices.
1 0 0 1

 
 3 1 2 6  1 2 3 −1
A=  and B = 3 6 9 −3
  
−1 2 5 −4

2 4 6 −2
2 3 7 2
Solution:
1 0 0 1 1 0 0 1 1 0 0 1
     
R2 → R2 − 3R1
 R3 → R3 − 2R2
 3 1 2 6 0 1 2 3 0 1 2 3
A=  R → R3 + R1
    
−1 2 5 −4 3 0 2 5 −3 R4 → R4 − 3R2 0 0 1 −9
   
R4 → R4 − 2R1
2 3 7 2 0 3 7 0 0 0 1 −9
1 0 0 1 1 0 0 1
   
0 1 2 3 0 1 0 21 
R4 → R4 − R3 

 R → R2 − 2R3 
  
0 0 1 −9 2 0 0 1 −9

0 0 0 0 0 0 0 0

Hence, rank(A) = 3 by using elementary row operations.


   
1 2 3 −1 1 2 3 −1
 R2 → R2 − 3R1 
B = 3 6 9 −3

0 0 0 0  .

R3 → R3 − 2R1
2 4 6 −2 0 0 0 0
Therefore, rank(B) = 1

Exercise 1.2

1. Transform the following matrix in to reduced row echelon form and determine the
rank of the following matrices.

2 1 −1 3
 
 
10 −8 0  1 −1 2 1
(a) A =  1 3 −5 , B = 
   
−4 6 −7 1

7 0 9
2 0 1 3
 
2 6 7 9
2 1 −1 3
 
 3 4 5 −1
 
   1 −1 2 1
 1
(b) C =  2 3 1 , D =  
−4 6 −7 1
  
 2 5 8 4
 
2 8 0 3
−1 2 2 10

1.4 Inverse of a matrix and its properties


Suppose A and B are square matrices of size n such that AB = In and BA = In . Then A is
invertible or non-singular and B is the inverse of A. In this situation, we write B = A−1 .
Notice that if B is the inverse of A, then we can just as easily say A is the inverse of B, or A
and B are inverses of each other.

13
Example 1.22
" # " #
1 1 2 −1
Show that B = is an inverse for the matrix A = :
1 2 −1 1
Solution:-By the definition there are two multiplications to confirm. (We will show later
that this isn’t necessary, but right now we are working strictly from the definition.) We
have
" #" #
2 −1 1 1
AB =
−1 1 1 2
" #
2(2) + (−1)1 2(1) + (−1)2
=
(−1)1 + 1(1) −1(1) + 1(2)
" #
1 0
=
0 1
= I2

and similarly
" #" #
1 1 2 −1
BA =
1 2 −1 1
" #
1(2) + 1(−1) 1(−1) + 1(1)
=
1(2) + 2(1) 1(−1) + 2(2)
" #
1 0
=
0 1
= I2

Therefore the definition for inverse is satisfied, so that A and B work as inverses to each
other.

Example 1.23
" #
1 1
Matrix A = cannot have an inverse.
1 1

Theorem 1.1
Suppose that A is invertible and that both B and C are inverses of A. Then B =C and
we will denote the inverse as A−1 .

Computing the Inverse of a Non Singular Matrix


Suppose A is a non singular square matrix of size n. Create the n × n matrix M by placing the
n × n identity matrix in to the right of the matrix A. Let N be a matrix that is row-equivalent
to M and in reduced row-echelon form then the first n columns of N is In and the last n
columns of N is A−1 .

14
Example 1.24
 
−7 6 12
Computing a Matrix Inverse of B =  5 5 7 
 

1 0 4
Solution:The augmented matrix is
 
−7 −6 −12 1 0 0
[B | I] =  5

5 7 0 1 0
1 0 4 0 0 1

by applying elementary row operation the equivalent reduced row echelon form is
 
1 0 0 −10 −12 9
0 1 0 13/2 8 11/2


0 0 1 5/2 3 5/2

So  
−10 −12 9
B −1 = 13/2 8 11/2
 

5/2 3 5/2

Properties of inverse matrix


Let A, B, C be matrices of the appropriate sizes so that the following multiplications make
sense, I a suitably sized identity matrix, and α a nonzero scalar. Then

1. (Uniqueness) The matrix A has at most one inverse, henceforth denoted as A−1 , provided
A is invertible.

2. (Double Inverse) If A is invertible, then (A−1 )−1 = A:

3. (2=3 Rule) If any two of the three matrices A, B and AB are invertible, then so is the
third, and moreover (AB)−1 = B −1 A−1 :

4. An is invertible and (An )−1 = (A−1 )n .

5. If A is invertible, then (αA)−1 = ( α1 )A−1 :

6. (Inverse/Transpose) If A is invertible, then (At )−1 = (A−1 )t .

7. (Cancellation) Suppose A is invertible. If AB = AC or BA = CA, then B = C:

15
1.5 Determinant of a matrix and its properties
Definition 1.12: (Determinant of a matrix)

Let A be an n × n matrix. Then the determinant of A denoted as det(A) or | A | is


defined recursively by:
If A = [a] is a 1 × 1 matrix, then det(A) = a. If A is a matrix of size n within ,n > 2
then

det(A) = A11 det(A11 ) − A12 det(A12 ) + A13 det(A13 ) − · · · + (−1)n+1 A1n det(A1n )
where A1j a sub matrix of A obtaining by deleting the fist row and the j th column.

So to compute the determinant of a 5 × 5 matrix we must build 5 sub matrices, each of size
4. To compute the determinants of each the 4 × 4 matrices we need to create 4 sub matrices
each, these now of size 3 and so on. To compute the determinant of a 10 × 10 matrix would
require computing the determinant of 10! = 10 × 9 × 8 × 7 × 6 × 5 × 4 × 3 × 2 = 3, 628, 800 1 × 1
matrices. Fortunately there are better ways. However this does suggest an excellent computer
programming exercise to write a recursive procedure to compute a determinant.
Lets compute the determinant of a reasonable sized matrixby hand.
3 2 −1
Suppose that we have the 3 × 3 matrix A =  4 1 6  then

−3 −1 2

det(A) =| A |
3 2 −1
= 4 1 6
−3 −1 2
1 6 4 6 4 1
=3 −2 −1
−1 2 −3 2 −3 −1
= 3(1 2 − 6 −1 ) − 2(4 2 − 6 −3 ) − (4 −1 − −3 )
= 3(1(2) − 6(−1)) − 2(4(2) − 6(−3)) − (4(−1) − (−3))
= 24 − 52 + 1
= −27

Theorem 1.2: (Exchanging Columns Changes the Sign of a Determinant).

If the matrix A0 is obtained from A by interchanging any two columns, and their deter-
minants exist, then |A0 | = −|A|.

The rule of Sarrus is a mnemonic for the 3 × 3 matrix determinant: the sum of the
products of three diagonal north-west to south-east lines of matrix elements, minus the sum of
the products of three diagonal south-west to north-east lines of elements, when the copies of
the first two columns of the matrix are written beside it as in the illustration. This scheme for
calculating the determinant of a 3 × 3 matrix does not carry over into higher dimensions.

16
Properties of determinants of matrix
1. If A is a triangular matrix, then the determinant of A is the product of all the diagonal
elements of A.
2. If B is obtained from A by multiplying one row of A by the scalar α, then det(B) =
α(det(A)).
3. If B is obtained from A by adding a multiple of one row of A to another row of A, then
det(B) = det(A).
1
4. The matrix A is invertible if and only if det(A) 6= 0 and det(A−1 ) = .
det(A)
5. The determinant of a product of two matrices is the product of their determinants. That
is,
det(AB) = det(A) det(B) =⇒ det(An ) = (det(A))n
6. If B is the transpose of a matrix A, then det(B) = det(A)

Minor In a Matrix
Suppose A is an n × n matrix and Aij is the (n − 1) × (n − 1) sub matrix formed by removing
row i and column j. Then the minor for A at location i j is the determinant of the sub matrix,
Mij (A) = det(Aij ).

Co factor In a Matrix
Suppose A is an n × n matrix and Aij is the (n − 1) × (n − 1) sub matrix formed by removing
row i and column j. Then the Co factor for A at location ij is the determinant of the sub
matrix, Cij (A) = (−1)i+j det(Aij ) .
Definition 1.13: (Adjoint)

If A = (aij ) is an n × n matrix, the adjoint of A, denoted by adj A, is the transpose of


the matrix of cofactors.

Hence  
c11 c21 c31 · · · cn1
 c12 c22 c32 · · · cn2
 

 
c
adjA =  13 c23 c33 · · · cn3 

 . .. .. . . ..
 ..

 . . . . 

c1n c2n c3n · · · cnn

17
Theorem 1.3
Let A be an n × n matrix. Then

A(adj A) = (det A)In = (adj A)A.

Note If the det(A) 6= 0, then A−1 = 1


det(A)
adj(A)
Example 1.25
 
2 3 −1
Let 0 −4 2 

.
1 −1 5
1. Determine

(a) The minors of all elements A. (c) The adj(A).


(b) The co factors of all elements of A. (d) The inverse of A.

Solution: Exercise

1.6 Solving system of linear equations


Definition 1.14
A linear equation in the variables x1 , x2 , · · · , xn is an equation of the form a1 x1 +
a2 x2 + · · · + an xn = b where the coefficients a1 , a2 , · · · , an and right hand side constant
term b are given constants.

Definition 1.15
A general system of m linear equations with n unknowns can be written as

a11 x1 + a12 x2 + ···+ a1n xn = b1


a21 x1 + a22 x2 + ···+ a2n xn = b2
.. .. .. .. (1.1)
. . . .
am1 x1 + am2 x2 + ···+ amn xn = bm .

Here x1 , x2 , . . . , xn , are the unknowns, a11 , a12 , . . . , amn are the coefficients of the system,
and b1 , b2 , . . . , bm are the constant terms.

A solution of a linear equation is any n-tuple of values (s1 , s2 , ...., sn ) which satisfies the
linear equation. For example, (−1, −1) is a solution of the linear equation x + 3y = −4
since−1 + (3 × −1) = −1 + (−3) = −4 , but (1, 5) is not.

Similarly, a solution to a linear system is any n-tuple of values (s1 , s2 , ...., sn ) which simulta-
neously satisfies all the linear equations given in the system.

18
For example,
3x + 2y − z = 1
2x − 2y + 4z = −2
−x + 12 y − z = 0
has as its solution (1, −2, −2) . This can also be written as:

x= 1
y =−2
z =−2

We also refer to the collection of all possible solutions as the solution set.

In general, for any linear system of equations there are three possibilities regarding solutions:

1. A unique solution In this case only one specific solution set exists. Geometrically this
implies the n-planes specified by each equation of the linear system all intersect at a
unique point in the space that is specified by the variables of the system.

2. No solution: The equations are termed inconsistent and specify n-planes in space which
do not intersect or overlap. It is not possible to specify a solution set that satisfies all
equations of the system.

3. An infinite range of solutions: The equations specify n-planes whose intersection is


an m-plane where m ≤ n. This being the case, it is possible to show that an infinite set
of solutions within a specific range exists that satisfy the set of linear equations.

Example 1.26
Given the system of linear equations,

x1 + 2x2 + x4 = 7
x1 + x2 + x3 − x4 = 3
3x1 + x2 + 5x3 − 7x4 = 1

we have n = 4 variables and m = 3 equations. Also,

a11 = 1a12 = 2 a13 = 0a14 = 1 b1 = 7


a21 = 1a22 = 1 a23 = 1a24 = −1 b2 = 3
a31 = 3a32 = 1 a33 = 5a34 = −7 b3 = 1

Additionally, convince yourself that x1 = −2, x2 = 4, x3 = 2, x4 = 1 is one solution


(but it is not the only one!).

Note that the above system can be written concisely as

···
    
a11 a12 a1n x1 b1
 a21

a22 ··· a2n  x
 
  2  b2 
 
 . .. ..   . = . 
 . ...
 . . .   ..   .. 
   

am1 am2 · · · amn xn bm

19
we may write the above simultaneous equations as

AX = b

where

···
     
a11 a12 a1n x1 b1
 a21

a22 ··· a2n 
  x2 
 
 b2 
 
A=
 .. .. ... ,
..   ..  and b =  .. 
X=   
 . . .   .   . 
am1 am2 · · · amn xm bm

A matrix A is called the coefficient matrix of the system, while the matrix

···
 
a11 a12 a1n b1
 a21

a22 ··· a2n b2 
 . .. .. .. 
 . ...
 . . . . 

am1 am2 · · · amn bm


obtained by adjoining b to A is called the augmented matrix of the system.
Remark: If bi = 0, ∀i = 1, 2, · · · , m then the linear system is called Homogenous otherwise,
non Homogenous.
Theorem 1.4
If[A|b] and [C|d] are row equivalent, then the systems AX = b and CX = d have exactly
the same solutions.

Remark: If A is an m by n matrix then the linear system AX = 0 has trivial solution


X = 0.
Theorem 1.5
If A is an m by n matrix then the equation AX = 0 has non trivial solution only, when
Rank(A) < n otherwise if Rank(A) = n then the trivial solution is unique.

The system of equation AX = b can be directly solved in the following cases.

1. If A = D, the equation (1.1) become

a11 x1 = b1
a22 x2 = b2
... ..
.
ann xn = bn
bi
The solution is given by xi = , aii 6= 0
aii
2. If A = L, the equation (1.1) become

a11 x1 = b1
a21 x1 +a22 x2 = b2
.. ..
. .
an1 x1 +an2 + · · · + ann xn = bn

20
Solving the first equation and then successively solving the 2nd, 3rd and so on.
b1 b2 − a21 x1 b2 − (an1 x1 + an2 + · · · + an(n−1) xn )
We obtain x1 = , x2 = , · · · , xn = ,
a11 a22 ann
where aii 6= 0, i = 1, 2, · · · , n
This method of solving equation is called forward substitution method.
3. If A = U , the equation (1.1) become

a11 x1 +a12 x2 +a13 x3 + · · · +a1n xn = b1


a22 x2 +a23 x3 + · · · +a2n xn = b2
.. ..
. .
ann xn = bn
Solving for the unknowns in the in the order xn , xn−1 , · · · , x1 , we get
bn bn−1 − a(n−1)nxn b1 − (a12 x2 + a13 x3 + · · · + a1n xn )
xn = , xn−1 = , · · · , x1 =
ann a(n−1)(n−1) a11
This method of solving equation is called backward substitution method. Therefore,
matrix A is solvable if it can be transformed in to any one of the forms D, U, L.

Theorem 1.6
Consider an m equations with n variables AX = b then

a) If rank(A|b) = rank(A) = n then the system has unique solution

b) If rank(A|b) = rank(A) < n then the system has infinitely many solutions.

c) If rank(A|b) > n then the system has no solution.

To solve a linear system , we have the following Methods;

1.6.1 Cramer’s rule


If AX = b is a linear system of n equations in n unknowns, and if detA 6= 0, then the system
| Ai |
has unique solution which can be determined by. xi = , i = 1, 2, · · · , n
|A|
Where Ai is the matrix obtained from A when ith column of A is replaced by b.
Consider the linear system (
a1 x + b 1 y = c 1
a2 x + b 2 y = c 2
which in matrix format is
" #" # " #
a1 b 1 x c
= 1 .
a2 b 2 y c2
Assume a1 b2 − b1 a2 nonzero. Then, with help of determinants x and y can be found with
Cramer’s rule as
c 1 b1 a1 c 1
c 2 b2 c 1 b2 − b1 c 2 a2 c 2 a1 c 2 − c 1 a2
x= = , y= = .
a1 b 1 a1 b2 − b1 a2 a1 b 1 a1 b 2 − b 1 a2
a2 b 2 a2 b 2

21
The rules for 3 × 3 matrices are similar. Given

 a1 x + b 1 y
 + c1 z = d 1
a2 x + b 2 y + c 2 z = d 2


a3 x + b 3 y + c 3 z = d 3

which in matrix format is     


a1 b 1 c 1 x d1
a2 b2 c2 y  = d2 .
    

a3 b 3 c 3 z d3
Then the values of x, y and z can be found as follows:

d1 b1 c1 a1 d1 c1 a1 b1 d1
d2 b2 c2 a2 d2 c2 a2 b2 d2
d3 b3 c3 a3 d3 c3 a3 b3 d3
x= , y= , and z = .
a1 b1 c1 a1 b1 c1 a1 b1 c1
a2 b2 c2 a2 b2 c2 a2 b2 c2
a3 b3 c3 a3 b3 c3 a3 b3 c3

Example 1.27
Solve the following system by cramer’s rule.

2x1 + 3x2 + 4x3 = 19


x1 + 2x2 + x3 = 4
3x1 − x2 + x3 = 9

Solution: The coefficient matrix is


 
2 3 4
1 2 1
A= 

3 −1 1

and column matrix  


19
b=4
 

9
, then
2 3 4
det(A) = 1 2 1 = 4 + 9 − 4 − 24 − 3 + 2 = −16 6= 0
3 −1 1
then the system has unique solution.

22
Example: Cont.

 
19 3 4 19 3 4
A1 =  4 2 1 & det(A1 ) = 4 2 1 = 38 + 27 − 16 − 72 − 12 + 19 = −16
 

9 −1 1 9 −1 1
 
2 19 4 2 19 4
A2 = 1 4 1 & det(A2 = 1 4 1 = 8 + 57 + 36 − 48 − 19 − 18 = 16
 

3 9 1 3 9 1
 
2 3 19 2 3 19
A3 = 1 2 4  & det(A3 ) = 1 2 4 = 36 + 36 − 19 − 114 − 27 + 8 = −80
 

3 −1 9 3 −1 9

det(A1 ) −16
∴ x1 = = =1
det(A) −16
det(A2 ) 16
x2 = = = −1
det(A) −16
det(A3 ) −80
x3 = = = 5.
det(A) −16

This is the solution of the system.

1. Problem 1: Use Cramer’s Rule to solve each for each of the variables.
x − y = 4
−x + 2y = −7
−2x + y = −2
x − 2y = −2
2. Problem 2: Use Cramer’s Rule to solve this system for z.

2x + y + z = 1
3x + z = 4
x − y − z = 2

1.6.2 Gaussian elimination method


Gauss elimination method is used to solve system of linear equations. In this method the linear
system of equation is reduced to an upper triangular system by using successive elementary
row operations. Finally we solve the value variables by using back ward substitution method.
This method will be fail if any of the pivot element aii , i = 1, 2, · · · , n becomes zero. In
such case we re-write equation in such manner so that pivots are non zero. This procedure is
called pivoting.
Consider system AX = b

Step 1: Form the augmented matrix [A|b]

Step 2: Transform [A|b] to row echelon form [U |d] using row operations.

Step 3: Solve the system U X = d by back substitution.

23
Example 1.28
Solve the following system using Gauss elimination method.

2x1 − 3x2 + x3 = 5
4x1 + 14x2 + 12x3 = 10
6x1 + x2 + 5x3 = 9

Solution: The augmented matrix of the system is


 
2 −3 1 5
4 14 12 10
 

6 1 5 9

Applying, elementary row operations on this matrix to change into its echelon form.
   
2 −3 1 5 2 −3 1 5
 R2 −→ R2 − 2R1 
4 14 12 10 0 20 10 0 
 
R3 −→ R3 − 3R1
6 1 5 9 0 10 2 −6
 
2 −3 1 5
R3 −→ R3 − 1/2R2 0 20 10 0

 
0 0 −3 −6

Since rank(A) = rank(A) = 3 = n the solution exists and is unique.


¯
2x1 −3x2 +x3 =5
20x2 +10x3 = 0
−3x3 = −6

From this we get x3 = 2. And using back substitution we have x2 = −1 and x1 = 0


Hence (0, −1, 2) is the solution of the system.

1.6.3 Inverse matrix method


Let AX = b is a system of n linear equations with n unknowns and A is invertible, then the
system has unique solution given by inversion method X = A−1 b.
Note:- When A is not square or is singular, the system may not have a solution or may have
more than one solution.

24
Example 1.29
Use the inverse of the coefficient matrix to solve the following system

3x1 + x2 = 6
−x1 + 2x2 + 2x3 = −7
5x1 − x3 = 10

Solution: Okay, let’s first write down the matrix form of this system.
    
3 9 0 x1 6
−1 2 2  x2  = −7
    

5 0 −1 x3 10

Now, we found the inverse of the coefficient matrix by using methods of finding Inverses
and is the following;

 
3 1 0
A = −1 2 2 


5 0 −1
 
−2 9 −10
=⇒ CA =  1 3 5 


2 −6 7
 
2 −1 2
=⇒ adj(A) =  9 −3 −6


−10 5 7

and det(A) = 3(−2) + 1(9) + 0(−10) = −6 + 9 = 3, then


   
2 −1 2 2/3 −1/3 2/3
A−1 = 1/3  9 −3 −6 =  3 −1 −2 
  

−10 5 7 −10/3 5/3 7/3
      
x1 2/3 −1/3 2/3 6 1/3
∴ x2  =  3
  
−1 −2  −7 =  5 
  

x3 −10/3 5/3 7/3 10 −25/3
Now each of the entries of X are x1 = 1/3, x2 = 5 and x3 = −25/3

25
Exercise 1.3

1. Solve the following linear system of equation by using Cramer’s rule, Gaussian
elimination method, and inverse method.

2x1 + 5x2 + 3x3 = 9 x+z =1 x + 2y + z = 3


(a) 3x1 + x2 + 2x3 = 3 (b) 2x + y + z = 0 (c) 2x + 5y − z = −4
x1 + 2x2 − x3 = 6 x + y + 2z = 1 3x − 2y − z = 5
2. Use rank of matrix to determine the values of a, b and c so that the following system
has:

(a) no solution (b) more than one solu- (c) a unique solution and
tion solve it.
1x + y − bz = 1 x + 2y − 3z = a x − 2y + bz = 3
i) 2x + 3y + az = 3 ii) 2x + 6y − 11z = b iii) ax + 2z = 2
x + ay + 3z = 2 x − 2y + 7z = c 5x + 2y = 2

1.7 Eigenvalues and Eigenvectors


Definition 1.16: (Eigenvalue, eigenvector)

Let A be a square matrix. Then if λA, is a real number and X a non zero column vector
satisfying AX = λX, we call X an eigenvector of A, while λ is called an eigenvalue of A.
We also say that X is an eigenvector corresponding to the eigenvalue λ.

Example 1.30
   
3 −2 0 1
Let A = −2 3 0, then show that X = 1 is eigenvector of A with λ = 1.
   

0 0 5 0

If λ is an eigenvalue of an n×n matrix A, with corresponding eigenvector X, then (A−λIn )X =


0, with X 6= 0, so det(A − λIn ) = 0 and there are at most n distinct eigenvalues of A.
Conversely if det(A − λIn ) = 0, then (A − λIn )X = 0 has a nontrivial solution X.

The equation det(A−λIn ) = 0 is called the characteristic equation of A, while the polynomial
det(A − λIn ) is called the characteristic polynomial of A. The characteristic polynomial of
A is often denoted by chA (λ).
Hence the eigenvalues of A are the roots of the characteristic polynomial of A.
Example 1.31
Find the eigenvalues for the matrix

a11 a12 a13 a14


 
 0 a22 a23 a24 
A=
 
0 0 a33 a34

 
0 0 0 a44

26
Theorem 1.7
if A is an n × n triangular matrix (upper triangular, lower triangular, or diagonal), then
the eigenvalues of A are the entries on the main diagonal of A.

Example 1.32
" #
2 1
Find the eigenvalues of A = and find all eigenvectors.
1 2
Solution: The characteristic equation of A is λ2 − 4λ + 3 = 0, or (λ − 1)(λ − 3) = 0
Hence λ = 1 or λ = 3. The eigenvector equation (A − In )X = 0 reduces to
" #" # " #
2−λ 1 x 0
=
1 2−λ y 0
or

(2 − λ)x + y = 0
x + (2 − λ)y = 0

Taking λ = 1 gives

x+y =0
x + y = 0.

which has solution x = −y, and let y = t is arbitrary non zero. Consequently the
eigenvectors corresponding to λ = 1 are the vectors
" # " #
−t −1
=t
t 1
" #
−1
with t 6= 0 which is the scalar multiple of .
1
Taking λ = 3 gives

−x + y = 0
x − y = 0.

which has solution x = y, and let y = t is arbitrary non zero. Consequently the eigen-
vectors corresponding to λ = 3 are the vectors
" # " #
t 1
=t
t 1
" #
1
with t 6= 0 hence the scalar multiple of .
1
" #
2 1
Therefore λ1 = 1 and λ2 = 3 are the eigenvalues of and the corresponding eigen-
1 2
" # " #
−1 1
vector are X1 = and X2 = respectively.
1 1

27
Theorem 1.8
If A is an n × n matrix, the following statements are equivalent.

(a) λ is an eigenvalue of A.

(b) The system of equations (A − λI)x = 0 has nontrivial solutions.

(c) There is a nonzero vector x such that Ax = λx.

(d) λ is a solution of the characteristic equation det(A − λI) = 0.

1.7.1 Diagonalization
Problem 1 Given an n × n matrix A, does there exist an invertible matrix P such that

P −1 AP

is diagonal?

Problem 2 Given an n × n matrix A, does A have n linearly independent eigenvectors?

Theorem 1.9
Let A be an n × n matrix having distinct eigenvalues λ1 , λ2 , · · · , λn and corresponding
eigenvectors X1 , X2 , · · · Xn respectively. Let P be the matrix whose columns are
respectively X1 , X2 , · · · , Xn . Then P is non singular and
 
λ1 0 0 ··· 0
 0 λ2 0 ··· 0
 
 
−1
0
P AP =  0 λ3 ··· 0 
 . .. .. .. .. 
 .. . . . .
 
0 0 0 ··· λn

Definition 1.17
If A and B are square matrices, then we say that B is similar to A if there is an invertible
matrix P such that
B = P −1 AP.

Definition 1.18
A square matrix A is said to be diagonalizable if it is similar to some diagonal matrix.
In other words, A is diagonalizable if there exists an invertible matrix P such that P −1 AP
is diagonal. In this case the matrix P is said to diagonalize A.

Theorem 1.10
If A is an n × n matrix, the following statements are equivalent.

(a) A is diagonalizable.

(b) A has n linearly independent eigenvectors.

28
Procedure for Diagonalizing a Matrix
1. Confirm that the matrix is actually diagonalizable by finding n linearly independent
eigenvectors. One way to do this is by finding a basis for each eigenspace and merging
these basis vectors into a single set S. If this set has fewer than n vectors, then the matrix
is not diagonalizable.

2. Form the matrix


P = [p1 p2 · · · pn ]
that has the vectors in S as its column vectors.

3. The matrix P −1 AP will be diagonal and have the eigenvalues

λ1 , λ2 · · · , λn

corresponding to the eigenvectors

p1 , p2 , · · · , pn

as its successive diagonal entries.

Example 1.33
In each of the following, determine if the the matrix is diagonalizable
   
" # 3 0 0 −1 4 −2
2 0
A= A = 0 2 0 A = −3 4 0 
   
1 2
0 1 2 −3 1 3

29
Debre Berhan University
Applied Mathematics One
Lecture note: Chapter 3
”Mathematics is the most beautiful and most powerful creation of
the human spirit.”
Stefan Banach

Dejen Ketema
Department of Mathematics
[email protected]

March, 2020
Contents

1 Limit and Continuity 2


1.1 Definition of Limit . . . . . . . . . . . . . . . . . . . . . . . . . . . . . . . . . . 2
1.1.1 Basic limit theorems . . . . . . . . . . . . . . . . . . . . . . . . . . . . . 4
1.1.2 Direct Substitution Property . . . . . . . . . . . . . . . . . . . . . . . . . 5
1.1.3 One sided limits . . . . . . . . . . . . . . . . . . . . . . . . . . . . . . . . 8
1.1.4 Infinite limits, limit at infinity and asymptotes . . . . . . . . . . . . . . . 9
1.2 Continuity and One Sided Continuity . . . . . . . . . . . . . . . . . . . . . . . . 17
1.2.1 Discontinuities . . . . . . . . . . . . . . . . . . . . . . . . . . . . . . . . 18
1.2.2 Removable discontinuities . . . . . . . . . . . . . . . . . . . . . . . . . . 18
1.2.3 Jump discontinuities . . . . . . . . . . . . . . . . . . . . . . . . . . . . . 19
1.3 Limit Exercises . . . . . . . . . . . . . . . . . . . . . . . . . . . . . . . . . . . . 21

1
Chapter 1

Limit and Continuity

1.1 Definition of Limit


As the precise definition of a limit is a bit technical, it is easier to start with an informal defi-
nition; we’ll explain the formal definition later.

We suppose that a function f is defined for x near c (but we do not require that it be defined
when x = c).
Definition 1.1: (Informal definition of a limit)

We call L the limit of f (x) as x approaches c if f (x) becomes close to L when x is close
(but not equal) to c , and if there is no other value L0 with the same property.
When this holds we write
lim f (x) = L
x→c
or
f (x) → L as x → c

Notice that the definition of a limit is not concerned with the value of f (x) when x = c (which
may exist or may not). All we care about are the values of f (x) when x is close to c , on either
the left or the right (i.e. less or greater).
Definition 1.2
Let f be a function defined on some open interval that contains the number c, except
possibly at c itself. Then we say that the limit of f as x approaches c is L, and we write

lim f (x) = L
x→c

if for every number  > 0 there is a number δ > 0 such that

if 0 <| x − c |< δ then | f (x) − L |< 

Since | x − c | is the distance from x to c and | f (x) − L | is the distance from f (x) to L,and 
can be arbitrarily small, the definition of a limit can be expressed in words as follows:
lim f (x) = L means that the distance between f (x) and L can be made arbitrarily small by
x→c
taking the distance from x to a sufficiently small (but not 0’s).
Alternatively, lim f (x) = L means that the values of f (x) can be made as close as we please
x→c
to L by taking x close enough to c (but not equal to c). We can also reformulate definition in
terms of intervals by observing that the inequality | x − c |< δ is equivalent to −δ < x − c < δ

2
1.1. DEFINITION OF LIMIT DBU

Figure 1.1: The  − δ definition of the limit of f (x) as x approaches a

, which in turn can be written as c − δ < x < c + δ. Also 0 <| x − c | is true if and only if
x − c 6= 0 , that is, x 6= a. Similarly, the inequality | f (x) − L |<  is equivalent to the pair of
inequalities L −  < f (x) < L + . Therefore, in terms of intervals, definition can be stated as
follows:
lim f (x) means that for every  > 0 (no matter how  is small) we can find δ > 0 such that if x
x→c
lies in the open interval (c − δ, c + δ) and x 6= c, then f (x) lies in the open interval (L − , L + )
Example 1.1

Prove that lim (4x − 5) = 7.


x→3
Solution:-
Let  be a given positive number. We want to find a number δ such that
|(4x − 5) − 7| <  whenever 0 < |x − 3| < δ. But |(4x − 5) − 7| = |4x − 12| = 4|x − 3|.

Therefore, we want 4|x − 3| <  whenever |x − 3| < δ this implies |x − 3| < whenever
4

|x − 3| < δ This suggests that we should choose δ = .
4

c Dejen K. 2020 3
1.1. DEFINITION OF LIMIT DBU

Example 1.2

Using formal definition of limit prove that lim x2 = 4


x→2
Solution:
We must show that for any  > 0 there exists δ > 0 such that |x2 − 4| <  when ever
0 < |x − 2| < δ Factoring, we get |x2 − 4| = |x − 2||x + 2| we want to show that |x2 − 4|
is small when x is close to 2. To do this, we first find an upper bound for the factor
|x + 2|. If x is close to 2, we know that the factor |x − 2| is small, and that the factor
|x + 2| is close to 4. Because we are considering values of x close to 2s, we can concern
ourselves with only those values of x for which |x − 2| < 1; that is, we are requiring the
δ, for which we are looking, to be less than or equal to 1. The inequality |x − 2| < 1
is equivalent to −1 < x − 2 < 1 which is equivalent to 1 < x < 3 or, equivalently,
3 < x + 2 < 5. This means that if |x − 2| < 1, then 3 < |x + 2| < 5 therefore, we have
1
|x2 − 4| = |x − 2||x + 2| < 5|x − 2|. Now we want 5|x − 2| <  or, equivalently |x − 2| < .
5

Thus, if we choose δ to be the smaller of 1 and , then when ever |x − 2| < δ, it follows
5
1 
that |x − 2| <  and |x + 2| < 5 because this is true when |x − 2| < 1 and so |x2 − 4| < .
5 5
Therefore, we conclude that |x2 − 4| <  whenever 0 < |x − 2| < δ if δ is the smaller of
 
the two numbers 1 and , which we write as δ = min(1, ).
5 5

Exercise 1.1
Prove that
x2
a) lim (−2x + 7) = 1 b) lim = 0, c) lim (x2 ) = 9
x→3 x→0 x2 + 1 x→3
d) lim (x2 − 9) = 0, e) lim (x2 ) = 0 f ) lim (−bx + c) − ab + c
x→−3 x→0 x→a

1.1.1 Basic limit theorems


Theorem 1.1
Operational Identities for Limits Suppose that x→c
lim f (x) = L and x→c
lim g(x) = M and that
k is constant. Then

• lim k · f (x) = k · lim f (x) = k · L


x→c x→c
 
• x→c
lim f (x) + g(x) = x→c
lim f (x) + x→c
lim g(x) = L + M
 
• lim f (x) − g(x) = lim f (x) − lim g(x) = L − M
x→c x→c x→c
 
• x→c
lim f (x) · g(x) = x→c
lim f (x) · x→c
lim g(x) = L · M

f (x) lim f (x) L


• lim = x→c = provided M 6= 0
x→c g(x) lim g(x)
x→c
M

c Dejen K. 2020 4
1.1. DEFINITION OF LIMIT DBU

1.1.2 Direct Substitution Property


If f (x) is a polynomial or a rational function and a is in the domain of f (x) , then

lim f (x) = f (a)


x→a

Example 1.3

1. Evaluate the following limits and justify each step.

(a) lim 2x2 − 3x + 4


x→5
r
x
(b) lim 3
x→4 −7x + 1
Solution:

1. (a)

lim (2x2 − 3x + 4) = lim (2x2 ) − lim (−3x) + 4 (by rule 1 and 2)


x→5 x→5 x→5
2
= 2 lim x − 3 lim x + 4 (by rule 3 and 7)
x→5 x→5
2
= 2(5 ) − 3(5) + 4 (by rule 9)
= 50 − 15 + 4
= 39

(b)
s s
x x
lim 3
= 3
lim [ ] (by low 11)
x→4 −7x + 1 x→4 −7x + 1
s
3
limx→4 x
= (by low 5)
limx→4 (−7x + 1)
s
4
= 3 (by low 3,9 and 1)
−7(4) + 1
s
34
=
−27

3
4
=−
3

Theorem 1.2
If f (x) 6 g(x) when x is near a (except possibly at a ) and the limits of f and g both
exist as x approaches a, then
lim f (x) 6 x→a
x→a
lim g(x)

c Dejen K. 2020 5
1.1. DEFINITION OF LIMIT DBU

Theorem 1.3: Squeeze Theorem

If f (x) 6 g(x) 6 h(x) when x is near a (except possibly at a ) and

lim f (x) = lim h(x) = L then lim g(x) = L


x→a x→a x→a

which is sometimes called the Sandwich Theorem.

Example 1.4: S
1
ow that lim x2 sin = 0
x→0 x
Solution:First note that we cannot use
1 1
lim x2 sin = lim x2 · lim sin
x→0 x x→0 x→0 x
1
because lim sin does not exist. However, since
x→0 x
1
−1 ≤ sin ≤1
x
Multiply both side by x2 we get
1
−x2 ≤ x2 sin ≤ x2
x
We know that lim −x2 = 0 and lim x2 = 0 taking f (x) = −x2 , h(x) = x2 and g(x) =
x→0 x→0
2 1
x sin by the Squeeze theorem, we obtain
x
1
lim x2 sin =0
x→0 x

c Dejen K. 2020 6
1.1. DEFINITION OF LIMIT DBU

Example 1.5
Show that
sin θ
lim =1
θ
θ→0

Proof: To show that the limit is 1, we begin with positive values of θ less than π. Notice
that from the following figure

We have Area 4 OAP 6 Area sector OAP 6 Area 4 OAT . We can express these areas
in terms of θ as follows:
1 1 sin θ
Area 4 OAP = base × hieght = (1)(sin θ) =
2 2 2
1 2 1 2 θ
Area sector OAP = r θ = (1 )(θ) =
2 2 2
1 1 tan θ
area 4 OAT = base × hieght = (1)(tan θ) =
2 2 2
sin θ θ tan θ
Thus 6 6 .
2 2 2
2
This last inequality goes the same way if we divide all three terms by the number sin θ
which is positive since 0 < θ < π/2

θ 1
16 6
sin θ cos θ
Taking reciprocals reverses the inequalities:
sin θ cos θ
1> >
θ 1
cos θ
Since lim = 1 and lim 1 = 1 then by the Sandwich theorem gives
θ→0 1 θ→0

sin θ
lim =1
θ→0 θ

sin θ
Recall that sin θ and θ are both odd functions . Therefore,f (θ) = is an even function,
θ
with a graph symmetric about the y-axis (see the above figure). This symmetry implies
that the left-hand limit at 0 exists and has the same value as the right-hand limit:
sin θ sin θ
lim+ = 1 = lim−
θ→0 θ θ→0 θ
sin θ
So lim =1
θ→0 θ

c Dejen K. 2020 7
1.1. DEFINITION OF LIMIT DBU

1.1.3 One sided limits


Definition 1.3
Let f be a function which is defined at every number in some open interval (a, c). Then
the limit of f (x), as x approaches from the right a, is L, written

lim f (x) = L
x→a+

if for any  > 0, however small, there exists a δ > 0 such that |f (x) − L| <  whenever
0<x−a<δ

Example 1.6

using the definition to prove that lim+ x=0
x→0
Solution:

1. Guessing a value for δ. Let  be a given√positive number. Here a = 0 and L = 0 so


√ want to find a number δ such that | x − 0| <  whenever 0 < x − 0√< δ that is
we
x <  whenever 0 < x < δ or, squaring both sides of the inequality x < , we
get x < 2 whenever 0 < x < δ. This suggests that we should choose δ = 

2. √
Showing√ that this δ works.
√ Given  > 0, let δ = 2 . If 0 < x < δ, then x <
δ = √ 2 = . So | x − 0| < . According to the definition, this shows that
limx→0+ x = 0

Definition 1.4
Let f be a function which is defined at every number in some open interval (d, a). Then
the limit of f (x), as x approaches from the left a, is L, written

lim f (x) = L
x→a−

if for any  > 0, however small, there exists a δ > 0 such that
|f (x) − L| <  whenever −δ < x − a < 0

Example 1.7

Show that lim− 4−x=0
x→4
Solution:
√ For every  >√0 we need to √ find a δ > 0 such that if −δ < x − 4 < 0 then
| 4 − x − 0| < . Since | 4 − x − 0| = | 4 − x| <  ⇒ |4 − x| < 2 ⇒ −2 < (x − 4) <
2 ⇒ −2 < (x − 4) < 0 then if −δ < (x − 4) < 0 then −2 < (x − 4) < 0. This suggests
that we should choose δ = 2 . Therefore, we conclude that

lim− 4 − x = 0
x→4

Theorem 1.4
lim f (x) = L if and only if
x→c

lim f (x) = L = lim− f (x)


x→c+ x→c

c Dejen K. 2020 8
1.1. DEFINITION OF LIMIT DBU

Exercise 1.2
|x|
Show that a) lim |x| = 0 and limx→0 does not exist.
x→0 x

1.1.4 Infinite limits, limit at infinity and asymptotes


Definition 1.5
Let f (x) be a function defined on some open interval that contains the number a, except
possibly at a itself. Then

i lim+ f (x) = ∞ means that for every positive number M there is a positive number
x→a
δ such that f (x) > M whenever 0 < x − a < δ

ii lim− f (x) = ∞ means that for every positive number M there is a positive number
x→a
δ such that f (x) > M whenever −δ < x − a < 0

lim f (x) = ∞ means that for every positive number M there is a positive number
iii x→a
δ such that f (x) > M whenever |x − a| < δ

Example 1.8
1
By using the definition prove that limx→0 =∞
x2
Solution: Let M be given large number, we want to find a δ > 0 such that f (x) > M
1 1
when ever 0 < |x − 0| < δ or 2 > M when ever 0 < |x| < δ that is, x2 < when
x M
1
ever 0 < |x| < δ or |x| < √ when ever 0 < |x| < δ. This suggests that we should take
M
1 1
δ = √ . Therefore lim 2 = ∞.
M x→0 x

Definition 1.6
Let f (x) be a function defined on some open interval that contains the number a, except
possibly at a itself. Then

i lim+ f (x) = −∞ means that for every negative number N there is a positive number
x→a
δ such that f (x) < N whenever 0 < x − a < δ

ii lim− f (x) = −∞ means that for every negative number N there is a positive number
x→a
δ such that f (x) < N whenever −δ < x − a < 0

iii lim f (x) = −∞ means that for every negative number N there is a positive number
x→a
δ such that f (x) < N whenever 0 < |x − a| < δ

c Dejen K. 2020 9
1.1. DEFINITION OF LIMIT DBU

Example 1.9
1
Show that limx→0− = −∞
x
Solution: Let N be given negative small number, we want to find a δ > 0 such that
1
f (x) < N when ever −δ < x − 0 < 0 or < N when ever −δ < x − 0 < 0 that is,
x
1 1
x> when ever −δ < x < 0. This suggests that we should take δ = − . Therefore
N N
1
limx→0− = −∞.
x

Definition 1.7
We say that f (x) has the limit L as x approaches infinity and write

lim f (x) = L
x→∞

if, for every number  > 0 there exists a corresponding positive number M such that for
all x
x > M ⇒ |f (x) − L| < 

Example 1.10
Show that
1
lim =0
x→∞x
Solution: Let  > 0 be given. We must find a positive number M such that for all x
1 1
x > M ⇒ | − 0| = | | < 
x x
1 1
x > M ⇒ x > because x is positive . The implication will hold if M = or any larger
 
positive number. This proves that
1
lim =0
x→∞ x

Definition 1.8
We say that f (x) has the limit L as x approaches negative infinity and write
if, for every number  > 0 there exists a corresponding negative number N such that for
all x
x < N ⇒ |f (x) − L| < 

c Dejen K. 2020 10
1.1. DEFINITION OF LIMIT DBU

Example 1.11
Show that
1
lim =0
x→−∞ x

Solution: Let  > 0 be given. We must find a negative number N such that for all x
1 1
x < N ⇒ | − 0| = | | < 
x x
1 1
x < N ⇒ x < − because x is negative. The implication will hold if N = − or any
 
small number. This proves that
1
lim =0
x→−∞ x

Theorem 1.5
Suppose that c is a constant and the limits lim f (x) = L and lim g(x) = M exist.
x→±∞ x→±∞
Then

1. Sum rule lim [f (x) + g(x)] = lim f (x) + lim g(x) = L + M.


x→±∞ x→±∞ x→±∞

2. Difference rule lim [f (x) − g(x)] = lim f (x) − lim g(x) = L − M.


x→±∞ x→±∞ x→±∞

3. Constant Rule lim [cf (x)] = c lim f (x) = cL.


x→±∞ x→±∞

4. Product Rule lim [f (x)g(x)] = lim f (x) lim g(x) = LM.


x→±∞ x→±∞ x→±∞

f (x) limx→±∞ f (x) L


5. Quotient Rule lim [ ]= = if lim g(x) 6= 0.
x→±∞ g(x) lim g(x) M x→±∞
x→±∞

c Dejen K. 2020 11
1.1. DEFINITION OF LIMIT DBU

Example 1.12
Using the above theorem evaluate the following limits at infinity

4x2 + 2 x2 + 1
a). lim b). lim
x→∞ 3x + 1 x→∞ 2x − 3

Solution: a). b).


s
√ 1 1
x2 (4 + ) 2 x+
4x2 + 2 x2 x +1 x
lim = x→∞
lim 1 lim = x→∞
lim
x→∞ 3x + 1 x→∞ 2x − 3 3
x(3 + ) 2−
x x
1
s
1 lim x + x→∞
lim
(4 + 2 ) x→∞ x
x =
= x→∞
lim 3
1 lim 2 − lim
(3 + ) x→∞ x→∞ x
s x ∞+0
1 =
lim (4 + 2 ) 2−0
x→∞ x ∞
= 1 =
lim (3 + ) 2
s
x→∞ x =∞
1
(x→∞
lim 4 + x→∞
lim 2 )
x
= 1
( lim 3 + lim )
x→∞ x→∞ x
q
(4 + 0)
=
(3 + 0)
2
=
3

Definition 1.9
The line y = b is called a horizontal asymptote of the curve y = f (x) if either
lim f (x) = b or/and lim f (x) = b
x→∞ x→−∞

Example 1.13
1 1
Find lim and lim .
x→∞ x x→−∞ x
Solution: Observe that when is large, is small. For instance, In fact, by taking large
enough, we can make as close to 0 as we please. Therefore, according to Definition , we
have Similar reasoning shows that when x is large negative, 1/x.

Definition 1.10
A line x = a is a vertical asymptote of the graph of a function y = f (x) if either
lim+ f (x) = ±∞ or/and lim− f (x) = ±∞
x→a x→a

c Dejen K. 2020 12
1.1. DEFINITION OF LIMIT DBU

Example 1.14
2x 2x
Find lim− ( ) and lim+ ( ).
x→3 x−3 x→3 x − 3
Solution: If x is close to 3 but larger than 3, then the denominator x − 3 is a small
positive number and 2x is close to 6. So the quotient 2x/x − 3 is a large positive number.
2x
lim+ ( )=∞
x→3 x − 3

Likewise If x is close to 3 but smaller than 3, then the denominator x − 3 is a small


negative number and 2x is close to 6. So the quotient 2x/x−3 is a large negative number.
2x
lim− ( ) = −∞
x→3 x−3
2x
The graph of the curve y = is given in the following figure. The line is a vertical
x−3
asymptote.

Example 1.15
Find the vertical and horizontal asymptotes for the graph of
√ s
x2 + 2 f (x) = 2x + 1 −
b). √ 3 x2 + 3
a). f (x) = c). f (x) =
x−1 4x2 + 5 27x2 − 1
Solution: a) We are interested in the behavior as x → ±∞ and as x → 1 where the
denominator is zero.
and then √ √ √
x2 + 2 x2 + 2 x2 + 2
lim− = −∞, lim+ = ∞, lim =1
x→1 x−1 x→1 x−1 x→∞ x − 1

and √
x2 + 2
lim = −1
x→−∞ x − 1

x2 + 2
∴ x = 1 is the vertical asymptote of f (x) = and also y = 1 and y = −1 are
√ x−1
x2 + 2
horizontal asymptote of f (x) = .
x−1

c Dejen K. 2020 13
1.1. DEFINITION OF LIMIT DBU

Special limit
1 − cos x • lim (1 + x) x = e
1
• lim =0 x→0
x→0 x
!mx n
k • lim √
n
=e
• lim 1+ = emk n→∞ n!
x→+∞ x s r
x
q √
1 n

• lim 1+ =e • n→∞
lim 2 2− 2+ 2 + ... + 2=π
x→+∞ x | {z
n
}
x
1 1
  x
a −1

• lim 1− = • lim = ln a, ∀a>0
x→+∞ x e x→0 x
x
x 1
   − 1
• lim = • lim 1 + a e−x − 1 x
= ea
x→+∞ x+k ek x→0

Exercise 1.3
Find the limit
 x x–1
4x tan x– sin x

6
1. lim 5. lim x3 9. lim 1 + 13. lim x+3
x→0 sin 3x x→0 x→∞ x x→∞ x–2
√ √
2. lim cos 3x– cos x
x2
6. lim 1–xcos x 10. lim x
1 + 3x
x→0 x→0+0 x→0
ln x– ln a
14. lim x–a
,
 n+5  x x→a
sin 5x– sin 3x 1 x+a
3. lim sin x 7. lim 1 + n
11. x→∞
lim x–a
x→0 n→∞
 3x  x 1
sin ax 1 x
4. lim 8. lim 1 + 12. lim . 15. lim (1 + sin x) x
x→0 sin bx x→∞ x x→∞ x+1 x→0

Solution:

1. 2. We factor the numerator:


4x
L = lim 3x–x 3x + x
sin 3x
x→0 cos 3x– cos x = –2 sin sin
3 · 4x 2 2
= lim = –2 sin x sin 2x.
x→0 3 sin 3x
4 3x
= lim This yields
3 x→0 sin 3x
4 1
= lim sin 3x cos 3x– cos x (–2 sin x sin 2x)
3 x→0 3x lim = lim
x→0 x 2 x→0 x2
lim 1
4 x→0
= . sin x sin 2x
3 lim sin3x3x = −2 lim · lim
x→0 x x→0 x
x→0
2 sin 2x
= −2 · 1 · lim
Since 3x → 0 as x → 0, we can write: 2x→0 2x
sin 2x
lim 1 = −2 · 2 lim = –4.
4 x→0 2x→0 2x
L=
3 lim sin3x3x
x→0
4 3. We use the following trigonometric iden-
=
3 lim sin3x3x tity:
3x→0
4 4
= = . x–y x+y
3·1 3 sin x– sin y = 2 sin cos .
2 2
c Dejen K. 2020 14
1.1. DEFINITION OF LIMIT DBU

Then we obtain Here


sin 5x– sin 3x 2 sin 5x–3x cos 5x+3x sin x
lim = lim 2 2 lim = 1 and lim cos x = 1.
x→0 sin x x→0 sin x x→0 x x→0
2 sin x cos 4x Hence,
= lim
x→0 sin x
= lim (2 cos 4x) . sin2 x2
x→0 L = 2 lim
x→0 x2
sin2 x2 4
!
As cos 4x is a continuous function at
x = 0, then = 2 lim ·
x→0 x2 4
2x
!
lim (2 cos 4x) = 2 lim cos 4x sin 2 1
x→0 x→0 = 2 lim x2
·
x→0 4
= 2 · cos (4 · 0) = 2 · 1 = 2. 4
1 sin2 x
4. = lim  22 .
2 x→0 x
2
sin ax
L = lim
x→0 sin bx x
! Here 2
→ 0 when x → 0 , therefore,
sin ax a bx
= lim · · 1 sin x2
!2
1 2 1
x→0 sin bx b ax L = xlim x = ·1 = .
sin ax bx a
!
2 2 →0 2
2 2
= lim · ·
x→0 ax sin bx b
6. Use the trigonometric formula:
lim sinaxax
a x→0 x
= . 1– cos x = 2sin2 .
b lim sinbxbx 2
x→0
Then the limit can be written in the form
Obviously, ax → 0 and bx → 0 as x → 0 √
. Then 1– cos x
L = lim
sin ax x→0+0 x
a lim ax a 1 a
L = x→0
q
sin bx = · = . 2sin2 x2
b lim b 1 b = lim
x→0 bx x→0+0 x
q
5. We apply the following transformations: √ sin2 x2
= 2 lim
tan x– sin x x→0+0 x
L = lim s
x→0 x3 √ sin2 x2
sin x
– sin x = 2 lim
= lim cos x 3 x→0+0 x2
x
s
x→0
 √ sin2 x2 4
sin x cos1 x –1 = 2 lim 2
·
= lim x→0+0 x 4
x3
v 
x→0
√ u sin2 x2
u
1
sin x (1– cos x) = 2 lim t x2 · √ 
= lim x→0+0 4
x→0 x3 cos x 4

sin x 1– cos x
 √ v
2 u sin2 x
u
= lim · 2 . 2
x→0 x x cos x = lim u t  2
2 x→0+0 x
As 1– cos x = 2sin2 x2 , we have v
2

2x
1 u lim sin
u
sin x 1– cos x
  u
2
L = lim · 2 =√ tx→0+0  2
x→0 x x cos x # 2 x
2
sin x 2sin2 x2
"
· 2
v"
= lim 1
u
sin x2
!#2
x→0 x x cos x = √
u
lim
t
x x
sin x sin2 x2 2 →0+0
lim · 2 lim 2 2
1 √ 2
2
x→0 x x→0 x 1
= . =√ · 1 =√ .
lim cos x
x→0 2 2

c Dejen K. 2020 15
1.1. DEFINITION OF LIMIT DBU

2a
We used here the fact that the limit re- Introduce the new variable: y = x–a
. As
mains the same when replacing x → 0 to x → ∞, y → 0 and, hence,
x
2
→0
2a 2a
x–a = , x=a+ .
7. y y
n+5
1

lim 1+ Substituting this into the function gives
n→∞ n
x
1 n
5 # 2a
" 
1
  
= lim 1+ 1+ L = lim 1+
n→∞ n n x→∞ x–a
2a

1
n 
1 5

= lim (1 + y)a+ y
= lim 1 + · lim 1 + y→0
n→∞ n n→∞ n 2a
= lim (1 + y)a · lim (1 + y) y
= e · 1 = e. y→0 y→0
2a 2a
=1·e =e .
8. By the product rule for limits, we obtain
3x
1 12. First we transform the base:

lim 1+
x→∞ x x x
 
x x x
1 1 1 L = lim
  
= lim · lim 1 +
1+ · lim 1+ x→∞ x + 1
x→∞ x x→∞ x x→∞ x 
x + 1–1 x

3
=e·e·e=e . = x→∞
lim
x+1
x
1

lim 1–
= x→∞ .
x+1
9. Substituting x6 = y1 , so that x = 6y and
1
y → ∞ as x → ∞ , we obtain Let – x+1 = y. Then x + 1 = – y1 , ⇒

6
x x = – y1 –1 and y → 0, if x → ∞. Now
lim 1+ we can find the limit:
x→∞ x
!6y x
1

1 L = lim 1–
= lim 1+ x→∞ x+1
y→∞ y 1
"
1
!y #6 = lim (1 + y)– y –1
y→0
= lim 1+ 1
y→∞ y lim (1 + y)– y
" !y #6 y→0
1 =
= y→∞
lim 1+ 6
=e . lim (1 + y)+1
y y→0
h 1 i–1
10. lim (1 + y) y
y→0
√ =
1
lim x 1 + 3x = lim (1 + 3x) x 1
–1
1

x→0 x→0 1
1 h 1 i3 = lim (1 + y) y = .
= lim (1 + 3x) 3x
·3
= lim (1 + 3x) 3x y→0 e
3x→0 3x→0
 3
1
= lim (1 + 3x) 3x = e3 . 13. We can transform this limit as follows:
3x→0
x + 3 x–1
 
11. We first transform the base of the func- L = lim
x→∞ x–2
tion: 
x–2 + 5 x–1


x+a x
 = x→∞
lim
L = lim x–2
x→∞ x–a 5 x–1
 

x–a + 2a x
 = x→∞
lim 1 +
= x→∞
lim x–2
x–a "  x–2 #
5(x–1)
x–2

2a x
 5 5

= lim 1 + . = lim 1+ .
x→∞ x–a x→∞ x–2

c Dejen K. 2020 16
1.2. CONTINUITY AND ONE SIDED CONTINUITY DBU

Replace the variable: 15. The limit can be represented in the fol-
5 5 5 lowing form:
= y, ⇒ x–2 = , ⇒ x = + 2.
x–2 y y 1
L = lim (1 + sin x) x
Here y → 0 as x → 0 . Then the limit is x→0
1 sin x
" 5(x–1)
 x–2 # x–2 = lim (1 + sin x) sin x · x
5 5 x→0
L = lim 1+ i sinx x
x→∞ x–2 h
= lim (1 + sin x) sin x
1
.
h 5
1 iy ( y +2–1)
x→0
= lim (1 + y) y
y→0 After taking logarithm, we have
h 1 i5+y
= lim (1 + y) y
1 i sinx x
!
y→0
h
1 i5 1 iy ln L = ln lim (1 + sin x) sin x
x→0
h h
= lim (1 + y) y · lim (1 + y) y
y→0 y→0 sin x h
 
1 i
h 1 i5 = lim ln (1 + sin x) sin x
= lim (1 + y) y · lim (1 + y) x→0 x
y→0 y→0 sin x  h 1 i
5
=e ·1=e . 5 = lim ·lim ln (1 + sin x) sin x .
x→0 x x→0

14. Let x–a = t . It is easy to see that t → 0 We notice that lim sinx x = 1 . Besides
x→0
as x → a . Then that, sin x → 0 as x → 0 , therefore,
ln x– ln a we can replace the transition x → 0 in
L = lim
x→a x–a the second limit with the equivalent limit
ln (t + a) – ln a sin x → 0 . This yields
= lim
t→0 t
1
ln t+a
 h i
= lim a ln L = 1 · lim ln (1 + sin x) sin x
sin x→0
t→0 t h 1 i
1 t

= ln lim (1 + sin x) sin x .
= lim ln 1 + . sin x→0
t→0 t a
1
Make one more change of variable: As lim (1 + sin x) sin x = e
sin x→0
t
= z, z → 0 as t → 0. ln L = ln e = 1.
a
Hence, the limit becomes
Thus, L = e
1 t
 
L = lim ln 1 +
t→0 t a
1
= lim ln (1 + z)
z→0 az
1 1
= lim ln (1 + z) z
a z→0
1

1
= ln lim (1 + z) z
a z→0
1 1
= ln e = .
a a

1.2 Continuity and One Sided Continuity


We are now ready to define the concept of a function being continuous. The idea is that we
want to say that a function is continuous if you can draw its graph without taking your pencil
off the page. But sometimes this will be true for some parts of a graph but not for others.
Therefore, we want to start by defining what it means for a function to be continuous at one

c Dejen K. 2020 17
1.2. CONTINUITY AND ONE SIDED CONTINUITY DBU

point. The definition is simple, now that we have the concept of limits:
Definition 1.11: (continuity at a point)

If f (x) is defined on an open interval containing c, then f (x) is said to be continuous at


c if and only if
lim f (x) = f (c).
x→c

Note that for f to be continuous at c, the definition in effect requires three conditions:

1. that f is defined at c , so f (c) exists,

2. the limit as x approaches c exists, and

3. the limit and f (c) are equal.

If any of these do not hold then f is not continuous at c.

The idea of the definition is that the point of the graph corresponding to c will be close to the
points of the graph corresponding to nearby x-values. Now we can define what it means for a
function to be continuous in general, not just at one point.
Definition 1.12
A function is said to be continuous on (a, b) if it is continuous at every point of the
interval (a, b) .

We often use the phrase ”the function is continuous” to mean that the function is continuous at
every real number. This would be the same as saying the function was continuous on (−∞, ∞),
but it is a bit more convenient to simply say ”continuous”.

Note that, by what we already know, the limit of a rational, exponential, trigonometric or
logarithmic function at a point is just its value at that point, so long as it’s defined there. So,
all such functions are continuous wherever they’re defined. (Of course, they can’t be continuous
where they’re not defined!)

1.2.1 Discontinuities
A discontinuity is a point where a function is not continuous. There are lots of possible ways
this could happen, of course. Here we’ll just discuss two simple ways.

1.2.2 Removable discontinuities


x2 − 9
The function f (x) = is not continuous at x = 3. It is discontinuous at that point be-
x−3
0
cause the fraction then becomes , which is undefined. Therefore the function fails the first
0
of our three conditions for continuity at the point 3, 3 is just not in its domain.

However, we say that this discontinuity is removable. This is because, if we modify the
function at that point, we can eliminate the discontinuity and make the function continu-
ous. To see how to make the function f (x) continuous, we have to simplify f (x) , getting
x2 − 9 (x + 3)(x − 3) x+3 x−3
f (x) = = = · . We can define a new function g(x)) where
x−3 (x − 3) 1 x−3

c Dejen K. 2020 18
1.2. CONTINUITY AND ONE SIDED CONTINUITY DBU

g(x) = x + 3 . Note that the function g(x)) is not the same as the original function f (x) ,
because g(x) is defined at x = 3 , while f (x) is not. Thus, g(x) is continuous at x = 3 , since
lim (x + 3) = 6 = g(3) . However, whenever x 6= 3 , f (x) = g(x); all we did to f to get g was
x→3
to make it defined at x = 3.

In fact, this kind of simplification is often possible with a discontinuity in a rational function.
We can divide the numerator and the denominator by a common factor (in our example x − 3
to get a function which is the same except where that common factor was 0 (in our example
at x = 3. This new function will be identical to the old except for being defined at new points
where previously we had division by 0.
x−3
However, this is not possible in every case. For example, the function f (x) = has
x2 − 6x + 9
a common factor of x − 3 in both the numerator and denominator, but when you simplify you
1
are left with g(x) = , which is still not defined at x = 3. In this case the domain of f (x)
x−3
and g(x) are the same, and they are equal everywhere they are defined, so they are in fact the
same function. The reason that g(x) differed from f (x) in the first example was because we
could take it to have a larger domain and not simply that the formulas defining f (x) and g(x)
were different.

1.2.3 Jump discontinuities


Illustration of a jump discontinuity

Not all discontinuities can be removed from a function. Consider this function:
( ot all discon-
1 if x > 0
tinuities can be removed from a function. Consider this function: k(x) = Since
−1 if x ≤ 0
lim k(x) does not exist, there is no way to redefine k at one point so that it will be continuous
x→0
at 0. These sorts of discontinuities are called nonremovable discontinuities.

Note, however, that both one-sided limits exist; lim− k(x) = −1 and lim+ k(x) = 1 . The
x→0 x→0
problem is that they are not equal, so the graph ”jumps” from one side of 0 to the other. In
such a case, we say the function has a jump discontinuity. (Note that a jump discontinuity is
a kind of nonremovable discontinuity.)

One-Sided Continuity
Just as a function can have a one-sided limit, a function can be continuous from a particular
side. For a function to be continuous at a point from a given side, we need the following three
conditions:

1. the function is defined at the point,

c Dejen K. 2020 19
1.2. CONTINUITY AND ONE SIDED CONTINUITY DBU

2. the function has a limit from that side at that point and
3. the one-sided limit equals the value of the function at the point.
A function f (x) is
• Left-continuous at x = c if lim− f (x) = f (c)
x→c

• Right-continuous at x = c if lim+ f (x) = f (c).


x→c
A function will be continuous at a point if and only if it is continuous from both sides at that
point. Now we can define what it means for a function to be continuous on a closed interval.
Definition 1.13
(continuity on a closed interval) A function is said to be continuous on [a, b] if and only
if

1. it is continuous on (a, b),

2. it is continuous from the right at a and

3. it is continuous from the left at b.

Notice that, if a function is continuous, then it is continuous on every closed interval con-
tained in its domain.

REMARK: The discontinuities in parts (b) and (c) are called removable discontinuities be-
cause we could remove them by redefining f at just the single number 0. The discontinuity in
part (d) is called jump discontinuity because the function ”jumps” from one value to another.
The discontinuities in parts (e) and (f) are called infinite or essential discontinuities.
Exercise 1.4

cos (2πx − a) , x ≤ −1
If the function f (x) = 3 is continuous, what is the value of a?
x + 1, x ≥ −1
Solution:
We calculate the left-hand and right-hand limits at x = −1 .

lim f (x) = lim cos (2πx–a)


x→–1–0 x→–1–0
= cos (–2π–a) = cos a,

The function will be continuous at x = −1 , if

lim f (x) = lim f (x) or cos a = 0.


x→–1–0 x→–1+0

Hence,
π
a= + πn, n ∈ Z.
2

c Dejen K. 2020 20
1.3. LIMIT EXERCISES DBU

Exercise 1.5

2
x

 + 2, x≤0
Let f (x) = ax + b,

0 ≤ x ≤ 1 Determine a and b so that the function f (x) is
2


3 + 2x–x , x ≥ 1
continuous everywhere.

Solution: Solution. The left-side limit at x = 0


 
lim f (x) = lim x2 + 2 = 2.
x→0–0 x→0–0

is Then the value of ax + b at x = 0 must be equal to 2.

ax + b = 2, ⇒ a · 0 + b = 2, ⇒ b = 2.

Similarly, the right-side limit at x = 1 is

lim f (x)
x→1+0
 
= lim 3 + 2x–x2 = 3 + 2–1 = 4.
x→1+0

As seen, the value of ax + 2 at x = 1 must be equal to 4.

ax + 2 = 4, ⇒ a · 1 + 2 = 4, ⇒ a = 2.

For given values of a and b , the function f (x) is continuous. The graph of the function

is sketched in Figure .

1.3 Limit Exercises


1. Evaluate lim (4x2 − 3x + 1)
x→2
One-Sided Limits
Since this is a polynomial, two can sim-
ply be plugged in. This results in Evaluate the following limits or state that the
4(4) − 2(3) + 1 = 16 − 6 + 1 = 11 limit does not exist.
x3 + x2
3. lim−
x→0 x3 + 2x2
x2 x + 1
Factor as 2 . In this form we can see that
x x+2
2. Evaluate lim (x2 ) 52 = 25 there is a removable discontinuity at x=0 and
x→5

c Dejen K. 2020 21
1.3. LIMIT EXERCISES DBU

1 x − 25
that the limit is 15. lim √
2 x→25
√x − 5 √
( x − 5)( x + 5)
4. lim |x2 + x| − x lim √
x→7− x→25 x−5 √
2 √
|7 + 7| − 7 = 49 = limx→25 x + 5) = 25 + 5) = 5 + 5 = 10
p |x|
5. lim − 1 − x2 16. lim
x→−1 x→0 x
|x|
p
2 2
p1 − x is defined if x < 1 , so the limit is lim
2
1−1 =0 x→0− x

p = limx→0− −x x = limx→0 −1 = −1

6. lim + 1 − x2 |x| x
x→−1 lim = lim+ = lim+ 1 = 1. The limit
p x→0+ x x→0 x x→0
1 − x2 is not defined if x2 > 1, so the limit does not exist.
does not exist. 1
17. lim
x→2 (x − 2)2
As x approaches 2, the denominator will be a
Two-Sided Limits very small positive number, so the whole frac-
tion will be a very large positive number. Thus,
Evaluate the following limits or state that the the limit is ∞.

limit does not exist. x2 + 16
18. lim
1 x→3 x−3
7. lim As x approaches 3, the numerator goes to 5
x→−1 x−1
1 and the denominator goes to 0. Depending on
− whether you approach 3 from the left or the
2
right, the denominator will be either a very
1 small negative number, or a very small positive
8. lim
x→4 x − 4 number. So the limit from the left is −∞ and
1 1
lim = −∞ lim+ = +∞ The limit the limit from the right is +∞. Thus, the limit
x→4− x − 4 x→4 x − 4
does not exist. does not exist.
3x2 − 8x − 3
1 19. lim
9. lim x→−2 2x2 − 18
x→2 x−2 2
3(−2) − 8(−2) − 3
1 1
lim− = −∞ lim+ = +∞.The limit 2(−2)2 − 18
x→2 x − 2 x→2 x − 2
3(4)+16−3
does not exist. = 2(4)−18 = 12+16−38−18
25
= −10 = − 52
x2 − 9 x2 + 2x + 1
10. lim 20. lim
x→−3 x + 3 x→2 x2 − 2x + 1
(x + 3)(x − 3) 22 + 2(2) + 1 4+4+1 9
= lim = lim x − 3 = = =9
x→−3 x+3 x→−3 22 − 2(2) + 1 4−4+1 1
= −3 − 3 = −6
x+3
x2 − 9 21. lim
x→3 x2 − 9
11. lim x+3 1 1
x→3 x − 3
lim = lim lim = −∞
(x − 3)(x + 3) x→3 (x + 3)(x − 3) x→3 x − 3 x→3− x − 3
lim = lim x + 3 = 3 + 3 = 6 1
x→3 x−3 x→3
lim = +∞. The limit does not exist.
x→3+ x − 3
x2 + 2x + 1
12. lim x+1
x→−1 x+1 22. lim
x→−1 x2 + x
(x + 1)(x + 1) x+1 1 1
lim lim = lim = = −1
x→−1 x+1 x→−1 x(x + 1) x→−1 x −1
= lim x + 1 = −1 + 1 = 0
x→−1
1
23. lim
x3 + 1 x→1 x2 + 1
13. lim 1 1 1
x→−1 x + 1
2
= =
(x2 − x + 1)(x + 1) 1 +1 1+1 2
lim 1
x→−1 x+1 24. lim x3 + 5x −
= lim x2 − x + 1 = (−1)2 − (−1) + 1 x→1 2−x
x→−1 3 1 1
=1+1+1=3 1 + 5(1) − =1+5− =6−1=5
2−1 1
x2 + 5x − 36 x2 − 1
14. lim 25. lim
x→4 x2 − 16 x→1 x2 + 2x − 3
(x − 4)(x + 9) x+9 4+9 13 (x − 1)(x + 1) x+1 1+1 2 1
lim = lim = = lim = lim = = =
x→4 (x − 4)(x + 4) x→4 x + 4 4+4 8 x→1 (x − 1)(x + 3) x→1 x + 3 1+3 4 2

c Dejen K. 2020 22
1.3. LIMIT EXERCISES DBU

5x x3 − 3x2 + 1
26. lim 35. lim
x→1 x2 + 2x − 3 x→−∞ 3x2 + x + 5
Notice that as x approaches 1, the numerator Top-heavy rational function, where the expo-
approaches 5 while the denominator approaches nent of the ratio of the leading terms is 1, so
0. However, if you approach from below, the de- the limit is −∞.
nominator is negative, and if you approach from
x2 + 2
above, the denominator is positive. So the lim- 36. lim
x→∞ x3 − 2
its from the left and right will be −∞ and +∞ Bottom-heavy, so the limit is 0.
respectively. Thus, the limit does not exist.

Limits of Piecewise Functions


Limits to Infinity Evaluate the following limits or state that the
limit does not exist.
Evaluate the following limits or state that the (
limit does not exist. (x − 2)2 if x < 2
37. Consider the function f (x) =
−x + π x−3 if x ≥ 2.
27. lim
x→∞ x2 + 3x + 2 (a) lim f (x)
This rational function is bottom-heavy, so the x→2−
limit is 0. (2 − 2)2 = 0
x2 + 2x + 1 (b) lim+ f (x)
28. lim x→2
x→−∞ 3x2 + 1 2 − 3 = −1
This rational function has evenly matched pow-
ers of x in the numerator and denominator, so (c) lim f (x)
x→2
the limit will be the ratio of the coefficients, i.e.
1 Since the limits from the left and right don’t
. match, the limit does not exist.
3
3x2 + x 38. Consider the function
29. lim
x→−∞ 2x2 − 15 
Balanced powers in the numerator and denomi- −2x + 1 if x ≤ 0

nator, so the limit is the ratio of the coefficients, g(x) = x + 1 if 0 < x < 4 .
3 
 2
i.e. . x +2 if x ≥ 4.
2
30. lim 3x2 − 2x + 1 (a) lim+ g(x)
x→4
x→−∞ 2
This is a top-heavy rational function, where the 4 + 2 = 16 + 2 = 18
exponent of the ratio of the leading terms is 2. (b) lim g(x) 4 + 1 = 5
Since it is even, the limit will be ∞. x→4−

(c) lim+ g(x) 0 + 1 = 1


2x2 − 32 x→0
31. lim 3
x→∞ x − 64 (d) lim g(x)
Bottom-heavy rational function, so the limit is x→0−
−2(0) + 1 = 1
0.
(e) lim g(x) Since the left and right limits
x→0
32. lim 6 match, the overall limit is also 1 .
x→∞
This is a rational function, as can be seen by
6x0 (f) lim g(x)
x→1
writing it in the form . Since the powers of
1x0 
x in the numerator and denominator are evenly 2x − 3
 if x < 2
matched, the limit will be the ratio of the coef- 39. Consider the function h(x) = 8 if x = 2
ficients, i.e. 6 . 
−x + 3 if x > 2.

3x2 + 4x
33. lim (a) lim h(x)
x→∞ x4 + 2 x→0
Bottom-heavy, so the limit is 0. 2(0) − 3 = −3

2x + 3x2 + 1 (b) lim h(x)


x→2−
34. lim
x→−∞ 2x2 + 3 2(2) − 3 = 4 − 3 = 1
Evenly matched highest powers of x in the nu-
(c) lim+ h(x)
merator and denominator, so the limit will be x→2
the ratio of the corresponding coefficients, i.e. −(2) + 3 = 1
3
. (d) lim h(x)
2 x→2

c Dejen K. 2020 23
1.3. LIMIT EXERCISES DBU

Since the limits from the right and left match,


the overall limit is 1 . Note that in this case, the
limit at 2 does not match the function value at
2, so the function is discontinuous at this point,
hence the function is non differentiable at this
point as well.

(a) At which points a in (0, 1, 2, 3, 4, 5) is g


continuous?
(b) At which points a in (0, 1, 2, 3, 4, 5) is g
continuous from the right?
(c) At which points a in (0, 1, 2, 3, 4, 5) is g
40. The graph of a function g is shown. continuous from the left?

c Dejen K. 2020 24

You might also like